Sei sulla pagina 1di 76

UJIAN BOARD KARDIOLOGI

15 JUNI 2015

Jumlahsoal : 150 Waktu : 100 menit


Tata caradanpetunjukujian
 Pilihjawaban yang paling benardanklik “Confirm Answer”, jawaban yang sudah di
“Confirm Answer” tidak bias diulangdandirubahjawabannya
 Jikamemilih “Skip”, soaltersebutakankembalisetelahandamenyelesaikansemuasoal
 Terdapatpetunjuk “COMPLETED 6 of 10”, angka6 menunjukkansoal yang telah di
“Confirm Answer” danangka10menunjukkanjumlahsoal
 Padabagiankananbawahterdapatsisawaktuujicobaujian
 Soalhanyadapatdikerjakanselamawaktuujian yang terteradiatas.
Setelahwaktuhabistidakdapatdilanjutkandenganalasanapapun
1. Mitral stenosis is considered severe when the :
a. Deceleration times >= 660 ms MVA:220/PHT. 759?DT
b. Mitral valve area is >= 1.5 cm2
c. No correct answer
d. Resting mean pressure gradient is >= 5 mmHg  Resting PAP>50
e. PHT is >= 200 msec

2. A-60-years-old male, dyspnea on exertion (NYHA III),


MI 2007; S/P stents to LAD, S/P ICD, JVP 4 cm; soft
PSM apex (Figure 1,2,3).
What is the ejection fraction?
a. 25-30 %
b. <10 %
c. 15-20 %
d. 20-25 %

3. If a drug is given that acts specially to increase myocardial contractility :


a. Stroke volume will decrease
b. Arterial pulse pressure will decrease
c. Mean arterial pressure will decrease
d. Capillary blood flow will decrease
e. Central venous pressure will decrease

4. A-50-years-old man had 3 graft CABG operation, LIMA to LAD, SVG to distal LCX,
and SVG to distal RCA with Aox time 90 minutes and CPB time 120 minutes. ECG
pre-op showed normal and went-well during operation. EF pre-op was 67% with
global normokinetic. On day 0 at ICU, the hemodynamic was unstable, lactate was
10, with mixed vein 45. After 3 days at ICU, patient was stable and plan move to
Intermediate wards but before moved patient had Atrial fibrillation.
What kind of medication that shoud be given to a post-op CABG patient to prevent
atrial fibrillation?
a. Ace-inhibitor
b. Statin
c. B-blocker  AHA guidelines recommendation IB (CABG)
d. Aspilet
e. All answer are correct

5. B-blocker may be even more widely used in the future because of their likely
ability to prevent which of these?
a. Hyperuricemia and gout
b. Control hypertension  MCQ 83
c. Myocardial infarction
d. Peripheral vascular disease
e. Peptic ulcer disease

6. A-50 years old man had 3 graft CABG operation, LIMA to LAD, SVG to distal LCX,
and SVG to distal RCA with Aox time 90 minutes and CPB time 120 minutes. ECG
pre-op showed normal and went-well during operation. EF pre-op was 67% with
global normokinetic. On day 0 at ICU, the hemodynamic was unstable, lactate was
10, with mixed vein 45. After 3 days at ICU, patient was stable and plan move to
Intermediate wards but before moved patient had Atrial fibrillation.
When should B-blocker be given to a post-op CABG patient to prevent atrial
fibrillation?
a. As soon as possible after extubation
b. Day 0  AHA recommendation IB : at least 24 hour before CABG
c. While the patient on ward just after rehabilitation
d. Day 1
e. While the patient move to IW
7. A following statement regarding the figure above are correct EXCEPT :
a. Spider view
b. An important view for the distal left
main stem and the proximal segments of
the LAD and circumflex
c. Position of the Image intensifier is left to
the midline and toward the feet of
the patient
d. A best view to assess the distal segment
of LAD and circumflex, the diagonal and
septal branch of LAD

8. A-45-years-old music teacher with a history of bicuspid aortic valve status post
aortic valve replacement is admitted to hospital with progressive fatigue and
dyspnea. She admit to non-adherence with sodium restriction, and has been
drinking 2-3 glasses of wine per night. Her bedside hemodynamic evaluation
suggest elevated right and left heart filling pressures and low cardiac output.an
echocardiogram reveals moderate LV dilation with an end-diastolic dimension of
6.4 cm, moderate systolic dysfunction with an EF of 30%, a well-seated mechanical
aortic valve with normal leaflet function, 3+ mitral regurgitation, and 2+ tricuspid
regurgitation. She respond well to empiric treatment with IV diuretics for 3 days,
with resolution of dyspnea. Her examination then reveals JVP of 6 cm of water, no
hepatomegaly, and warm extremities without edema. Plans are made for discharge
home with follow-up in the HF clinic in 1 week.
Which of the following is a equality measurement recommended by the ACCF/AHA
for all HF patients at the time of hospital discharge?
a. Documentation of LV size
b. Scheduling a follow-up visit within 1 week  AHA guideline 2013 HF, IIA
c. Optimizing therapy with an aldosterone antagonist
d. Counseling regarding alcohol cessation
e. Achieving optimal volume status  AHA 2013 HF: IB
9. A-68-years-old woman presents to ER with severe orthopnea for the past 12
hours. She has a history of hypertension, osteoarthritis and borderline diabetes
mellitus. Exam revealed irregular rate and rhythm with heart rate 115 bpm, BP
190/110 mmHg, venous pressure 14 cm, and BMI 38 kg/m2. Chest auscultation
showed bilateral rales, while precordial exam revealed an irregular and rapid
heart rate with distant heart sounds and no discernible gallop. The patient is given
IV furosemide, diltiazem and nitroglycerin infusions, with prompt improvement in
symptoms.Forty-eight hours later, she is now on oral losartan, diltiazem, aspirin
and furosemide. Her heart rate now is irregular at 70 bpm, BP is 130/65 mmHg,
venous pressure is normal, but she still complain moderate dyspnea on exertion.
Which of the following would you recommend?
a. Nocturnal supplemental oxygen
b. Switch diltiazem to atenolol
c. Transesophageal echocardiogram-guided cardioversion  New AF
d. Recommend bariatric surgery
e. Switch losartan to lisinopril

10. A-55-years-old male presents with shortness of breath for 6 months. He’s been
sleeping in a chair for the past months because he cough whenever he lies back.
He’s also noticed that his heart racing most of the time. He’s had no antecedent
illness. He tells you that his brother died suddenly several years ago in his 30s. He
drinks socially.
He appears comfortable with a BP of 110/75 and a regular HR of 115. His JVP is 18
cm of water, lungs are clear, and the heart is enlarged with palpation. He has a
gallop rhythm and a diffuse systolic precordial murmur. The abdomen is mildly
tender in the RUQ with liver edge easily palpable. He is warm with trace pedal
edema. An EKG demonstrate sinus tachycardia and narrow QRS.
In addition to diuretic management, you initiate :
a. Aldosterone antagonist
b. Digoxin
c. Angiotensin receptor blocker
d. Angiotensin converting enzyme (ACE) inhibitor  SDH JELAS
e. Beta-blocker
11. An active 40-years-old premenopausal woman is seen in the emergency
department because of substernal chest pressure. The pressure start during an
argument with her husband, decreased as she was transported to the hospital, and
was gone by the time she was examined. She has not had this symptom before. Her
rate is 90 bpm and BP is 150/90 mmHg. Finding on physical examination are
otherwise normal. The finding on ECG and chest radiograph are normal. Serum
creatin kinase-MB and troponin I levels are normal at baseline and 6 hours later.
Her only risk factor for CAD is mild obesity. Her BMI is 31.
Which of the following is the best course of action?
a. Discharge the patient and perform an exercise electrocardiogram within 72
hours  PTP Mod Low (MKSAP 13)
b. Discharge the patient and perform an exercise echocardiogram within 72
hours
c. Discharge the patient and recommend follow-up with her primary care
physician
d. Admit the patient to the cardiac care unit for monitoring
e. Discharge the patient and perform an exercise nuclear perfusion study within
72 hours

12. A 65-years-old man with a longstanding ischemic cardiomyopathy and LVEF 20%
by an echocardiogram obtained 1 month ago, was admitted to hospital yesterday
for increased shortness of breath over the last 3 weeks. You are evaluating him in
consultation with his internist. Overnight he had made 1 liter of urine in response
to diuretic therapy.
On examination, you find him to be in no acute distress at rest, BP 110/70 mmHg,
pulse 96 bpm and regular, and his BMI is 38 kg/m2. He reports persistent
shortness of breath when ambulating in the corridors and in the supine position,
the latter requiring two pillows to allow him to breathe comfortably. The
remainder of his examination is notable for JVP estimated to be 14 cm, S12with no
S3 or murmur, a benign abdomen with no hepatomegaly, and extremities which
are entirely free of edema and are warm to touch. His mental status is normal.
Laboratory studies today include : sodium 140, BUN 55, creatinine 1.9 (stable from
admission) and hematocrit 34. Admission BNP was 75. Chest x-ray shows
cardiomegaly with no evidence of pulmonary edema, pleural effusions, and
infiltrates.
At this point, which of the following is the most appropriate therapy to
recommend?
a. Administration of IV Lasix 80 mg bid for presumed acute on chronic systolic
failure
b. Administration of normal saline at 75 cc/hour for possible volume depletion
c. Order an echocardiogram and base further therapeutic decision about the
need for diuretics or saline administration on the E/e’ ratio  Insya allah
d. Schedule for a right heart catheterization and adjust therapy based on
invasively measured hemodynamics
e. Administration of IV Lasix 80 mg bid plus an infusion of dobutamine 2.5
mcg/kgBB/min to provide inotropic support for treatment of acute on
chronic systolic heart failure complicated by renal insufficiency

13. A 50-years-old male patient brought this treadmill test result that performed after
he had finished phase II of cardiac rehabilitation program after undergone elective
PCI. His baseline HR was 70 bpm. There was no ST-T changes, no arrhythmia,
normal response of BP and HR during exercise. The test was stopped at 9 minutes
of exercise (Bruce protocol), with HR 150 (Rate Perceived Exertion : Borg Scale
17). What is his target for treadmill at home using 65-75% HRR (Heart Rate
Reserve) formula?
a. 102-107 bpm
b. 97-112 bpm
c. 110-127 bpm
d. Can’t be measured
e. 122-130 bpm  Karvonen HRR

14. A 50-years-old African-American with no significant past medical history presents


with palpitations. On examination, he has a mildly elevated JVP at 10 cm H2O, a
faint parasternal lift, and an RV S3 gallop. His ECG is notable for sinus rhythm, with
incomplete RBBB. A Holter monitor reveals frequent runs of nonsustained
ventricular tachycardia with LBBB pattern. Coronary angiography reveals no
significant epicardial coronary artery disease. Echocardiogram suggest RV cavity
dilation and moderate tricuspid valve regurgitation.
This patient most likely has a disorder associated with a defect in the gene
encoding which of the following proteins?
a. Lamin A/C
b. Plakoglobin  BRAUNWALD 543
c. Tafazzin
d. Myosin heavy chain
e. Dystrophin

15. A 46-years-old man begin an exercise training program that involves jogging on
treadmill for 40 minutes three times per week. Before beginning this program, he
underwent exercise testing that showed a peak heart rate of 120/min and a peak
estimated metabolic equivalent level of 7. After 6 months of training, he undergoes
another treadmill test that shows a peak heart rate of 120/min and a peak
metabolic equivalent level of 9. Both tests were terminated because of patients
fatigue. He had no ischemic response on either test.
Which of the following best describes the results of this training program?
a. Achievement of a higher peak metabolic equivalent level is primarily caused
by a greater level of effort on the second test
b. The finding of no change in peak heart rate indicates that no training benefit
was achieved
c. The finding indicate that the patient did not exercise to exhaustion during the
first test
d. Change in the peak metabolic equivalent level are most likely caused by
changes in pulmonary ventilation that occur with exercise training
e. The heart rate response to submaximal levels of exercise is now lower 
MKSAP

16. A 50 year old woman has left shoulder discomfort that she notices when she
climbs stairs, walk uphill, or becomes upset with her children. The discomfort
generally resolves within one or two minutes after she stops the activity. Her
resting ECG showed nonspecific ST- and T-wave abnormalities, with less than
1mm of ST segment depression. The patient had a TMET and exercised for 6
minutes on a bruce protocol to a HR of 130 bpm and a BP of 155/70 mmHg. She
stopped because of severe chest heaviness and left shoulder pain. The exercise
ECG did not show any ST-segment depression.
Which of the following statements is correct about this patient’s Duke treadmill
score?
a. It is calculated as +6 and places the patient at low risk for subsequent cardiac
event
b. It is not clinically meaningful because the treadmill score applies only to men
c. It is not clinically meaningful because the treadmill score applies only to
patients with normal resting ECGs
d. It is calculated as -2 and places the patient at low-to-intermediate risk for
subsequent cardiac events
e. It is calculated as -2 and places the patient at intermediate risk for
subsequent cardiac event  MAYO

17. A 46-year-old woman with history of breast cancer in remission presents with a 2-
week history of dyspnea on exertion and weight gain. She does not have any
history of cardiac disease, but has a history of hypertension and type 2 diabetes
mellitus. She presents to the emergency room because she now has shortness of
breath at rest. She denies other cardiac complaints. On admission, her creatinine is
1.2 mg/dl, potassium is 4.4 mEq/L, HbA1c is 6.4%, blood glucose is 123 mg/dl,
low-density lipoprotein (LDL) 112, high-density lipoprotein (HDL) 32,
triglycerides 188mg/dl, and BNP is 2202 pg/ml. her ECG shows sinus tachycardia,
110 bpm, intraventricular conduction delay with a prolonged QRS interval of 128
ms. After intravenous diuretic therapy in the emergency room, she gets admitted
and an echocardiogram revealsa dilated LV of 7 cm, and LVEF of 10% with
spontaneous echocontrast in the LV. On day 3, she undergoes coronary
angiography, which reveals angiographically normal coronary arteries. An ACE
inhibir=tor and a beta blocker are started during the hospital course, and she
tolerates these well with a modest improvement in her symptoms.
Which of the following is the best additional treatment choice at this point?
a. Cardiac resynchronization therapy with defibrillator
b. Cardiac resynchronization therapy without defibrillator
c. Aldosterone antagonist  MKSAP 13
d. Aspirin
e. Statin

18. An 80 years-old hypertensive male referred to cardiology clinic with stable NYHA
fc III angina for 3 months treated with aspirin, metoprolol succinate 150 mg daily,
isosorbidemononitrate 120 mg daily, and simvastatin 40mg daily. On exam, the
heart rate is 57, the blood pressure is 98/60 mmHg, and the cardiopulmonary
exam is unremarkable. Resting ECG is within normal limits. An exercise stress test
is significant for 2 mm horizontal ST depression and exercise limiting chest
discomfort at 6 METs.
The duke treadmill score for this patient is:
a. -10, intermediate risk
b. -14, intermediate risk
c. -14, high risk  OK
d. -9, intermediate risk
e. -9, high risk

19. Randomized-controlled trial study that showed exercise training intervention after
coronary angioplasty can improve functional capacity and quality of life, lowered
residual stenosis, decreased events and hospital readmission significantly is …
a. ExTraMATCH
b. ETICA
c. EURO-ASPIRE
d. GOSPEL  after MI
e. HF-action
20. Each of the following statements regarding high-sensitivity C-reactive protein
(hsCRP) is true:
a. The cardiovascular benefit of aspirin therapy appears to be greatest in
patients with elevated hsCRP levels  BRAUNWALD
b. Included in global risk score, a predictive risk score tools to predict
cardiovascular mortality in asymptomatic patients
c. An hsCRP level > 2 mg/L in a patient with unstable angina is associated with
an increased with an increased risk of recurrent coronary events
d. Statins reduce hsCRP in a manner directly related to their low density
lipoprotein-lowering effect
e. An elevated level of hsCRP is predictive of the onset of metabolic syndrome

21. A 28-year-old woman eith aortic and mitral mechanical valve prostheses that were
placed for congenital heart disease comes to your office for a prepregnancy
consultation. She is active and asymptomatic. Physical examination shows normal
prosthetic, aortic, and mitral valve prosthesis auscultation. Results of laboratory
studies are normal, except for a therapeutic international normalized ratio on
warfarin anticoagulation.
Which of the following recommendations is appropriate for the management of
this patient during pregnancy?
a. Discontinue warfarin anticoagulation and initiate treatment with aspirin and
dypiridamole for the first trimester
b. Discontinue warfarin anticoagulation and initiate treatment with clopidogrel
and aspirin during the first trimester and resume treatment with warfarin for
the rest of the pregnancy until shortly before delivery
c. Discontinue warfarin administration and initiate treatment with enoxaparin,
30mg subcutaneously three time daily, plus aspirin
d. Continue warfarin administration throughout pregnancy and start heparin
5000 IU subcutaneously three times daily, plus aspirin
e. Discontinue warfarin and initiate dose adjusted unfractionated heparin
subcutaneously during the first trimester and resume treatment with
warfarin for the rest of the pregnancy until shortly before delivery MKSAP
13
22. Hydration prior to contrast angiography is useful to:
a. Reduce the chance of contrast nephropathy
b. Reduce the chance of pulmonary edema
c. Reduce the chance of thrombotic episodes in cyanotics
d. Reduce the chance of contrast nephropathy and reduce the chance of
thrombotic episodes in cyanotics  MCQ 338

23. A 45-year-old man is admitted to a hospital due to typical chest pain after exercise
with 4 hours onset. He reveals that this not the first time, the symptoms were
already develop over one year and he had a syncope last month. The patient was a
heavy smoker, his father already passed away at his 50’s due to heart disease. On
physical examination the blood pressure is 140/80 mmHg, normal S1 and S2 with
a grade 3/6 holosystolic murmur at the apex and axilla. An electrocardiogram
shows left ventricular hyperthropy with strain, left atrial enlargement. Cardiac
enzyme were normal.
This patient could possibly having :
a. Hypertrophic cardiomyopathy  sumber soal blm nemu
b. Amyloidosis
c. Farby disease
d. Aortic stenosis
e. Hypertensive heart disease

24. A 28-year-old woman is referred for evaluation of persistent dyspnea related to


mitral stenosis. The patient is 30 weeks pregnant, and her dyspnea persists
despite treatment with metoprolol, furosemide and digoxin. Her heart rate is
70/min. an echocardiogram shows severe mitral valve stenosis, with a mean
gradient of 14 mmHg and a mitral valve ares of 1 cm2. Trivial valve regurgitation is
noted. The estimated right ventricular systolic pressure is 50 mmHg. She has
crackles in both lung bases and bilateral lower extremity edema.
Which of the following interventions do you recommend?
a. Surgical mitral valvotomy
b. Transesophageal echocardiography followed by percutaneous mitral balloon
valvuloplasty  MKSAP 13
c. Diagnostic cardiac catheterization
d. Urgent delivery of the fetus followed by reassessment of maternal cardiac
status
e. Fetal echocardiogram

25. Which of the following would you recommend for a 19-year-old asymptomatic,
nonobstructive HCM patient with a maximal LV wall thickness 18 mm, who is
playing professional soccer?
a. Discontinuation from organized, competitive sports
b. EP study
c. ASA
d. Disopyramide

26. A 45-year-old man is admitted to a hospital due to typical chest pain after exercise
with 4 hours onset. He reveals that this not the first time, the symptoms were
already develop over one year and he had a syncope last month. The patient was a
heavy smoker, his father already passed away at his 50’s due to heart disease. On
physical examination the blood pressure is 140/80 mmHg, normal S1 and S2 with
a grade 3/6 holosystolic murmur at the apex and axilla. An electrocardiogram
shows left ventricular hypertrophy with strain, left atrial enlargement. Cardiac
enzyme were normal.
What is the best modalities to define the diagnosis of this patient?
a. Cardiac CT
b. Coronary Angiography
c. Transthoracic Echocardiography  sumber ?
d. Cardiac MRI
e. Treansoesophageal Echocardiography
27. Classification of myocardial infarction type 2 according to the universal definition
of myocardial infarction?
a. Sudden unexpected cardiac death
b. MI associated with percutaneous coronary intervention  tipe 4
c. MI associated with ischemia secondary to either increased oxygen demand or
decreased supply  ESC Guidelines
d. MI associated with coronary artery bypass graft surgery  tipe 5
e. Spontaneous MI result from a primary coronary event (coronary plaque
erosion, rupture or dissection)

28. A 50-year-old man had a 3 graft CABG operation, LIMA to LAD, SVG to distal LCx
and SVG to distal RCA with Aox time 90 minutes and CPB time 120 minutes. ECG
pre-op showed normal and went well during operation. EF pre-op was 67% with
global normokinetic. On day 0 at ICU, the hemodynamic was unstable, Lactate was
10, with mixed vein 45. After 3 days in ICU patient stable and plan move to
intermediate wards but before move patient had atrial fibrillation.
What are the possibilities happened on the patient on day 0 post op that made he
had unstable hemodynamic?
a. All the answers are correct  diskusi, ndak nemu sumber soalnya
b. Acute left heart failure
c. Bleeding
d. Volume depletion
e. Perioperative myocardial infarction

29. A 45-year-old man is admitted to a hospital due to typical chest pain after exercise
with 4 hours onset. He reveals that this not the first time, the symptoms were
already develop over one year and he had a syncope last month. The patient was a
heavy smoker, his father already passed away at his 50’s due to heart disease. On
physical examination the blood pressure is 140/80 mmHg, normal S1 and S2 with
a grade 3/6 holosystolic murmur at the apex and axilla. An electrocardiogram
shows left ventricular hypertrophy with strain, left atrial enlargement. Cardiac
enzyme were normal.
On imaging studies shows this following findings, EXCEPT?
a. Pulmonary venous systolic flow reversal
b. LVOT pressure gradient 35mmHg
c. Systolic anterior motion of the mitral leaflet
d. LV wall thickness is 32mm symmetrical

30. False statement regarding the assessment of coronary microvascular obstruction


by CMR:
a. A dark core of hypoenchancement within the infarct tissue is taken as a
region of coronary microvascular obstruction
b. Infarcted tissue is seen as a region of hypo enhancement
c. Images obtained 10-15 min afer injecting a gadolinium-based contrast 
Imaging book
d. The index of microcirculatory

31. During the cardiac catheterization of a person who was found to be normal, the
blood withdrawn through the catheter had an oxygen saturation of 60%, and the
recorded pressure oscillated each heartbeat between 14 and 26 mmHg. Most
likely, the catheter tip was located in the :
a. Right atrium
b. Foramen ovale
c. Coronary sinus
d. Azygos vein
e. Pulmonary Artery

32. Factors that will reduce cardiac output?


a. Volume load
b. Baroreceptor stimulation
c. Sympathetic tone
d. Digitalis glycoside
e. Phospodiesterase inhibitor (milrinone, theophyline)
33. Regarding stress echocardiography, which of the following statement is
appropriate to perform stress echocardiography:
a. Asymptomatic patient withAgatson score greater or equal to 400
b. Patient with chest pain with low pre-test probability of CAD and the ECG is
uninterpretable
c. Exercise stress in patient with history of non-sustained ventricular
tachycardia, Framingham risk is moderate to high
d. As a risk assessment following unstable angina/ non ST elevation MI without
sign and symptom of heart failure and not planning to undergo early
catheterization
e. All answer are correct  Guidelines stress echo ACCF/SCAI/SCMR

34. Regarding trans thoracal echocardiography, which of the following statement is


appropriate to perform trans thoracal echocardiography:
a. Routine surveillance (<1 y) or mild stenosis without change in clinical
status/cardiac exam
b. Initial evaluation in patient with murmur or click without other signs and
symptoms of structural heart disease
c. To evaluate arrhythmias in infrequent APC/VPC without evidence of heart
disease
d. Routine surveillance (>3y) of mild stenosis without change in clinical status/
cardiac exam  Guideline VHD : 2-3 thn surveilnce
e. Routine perioperative evaluation of cardiac structure and function prior to
non-cardiac solid organ transplantation

35. Regarding echocardiographic score used to predict outcome of mitral balloon


velocity, when the Mobility: mid and base portion of the leaflet mobile normally;
Subvalvular thickening: The thickening chordate structure extending up to one-
third of the cordallength; Thickening: The valve thickening through the entire
leaflet; and Calcification: some scattered areas of brightness confined to leaflet
margins, the mitral score will be:
a. 3-2-2-2
b. 2-3-3-2
c. 2-2-3-2
d. 2-2-2-2
e. 2-3-2-2

36. A 32 year-old woman with primary PH is referred to follow up of her pulmonary


pressures. In addition of tricuspid regurgitant jet velocity, the following
information is needed to estimate pulmonary systolic pressure in this patient:
a. Antegrade velocity in the pulmonary artery
b. Mitral regurgitation maximal velocity
c. Imaging of the inferior vena cava
d. Hepatic vein flow
e. RV free wall thickness

37. What is the diagnosis from the pressure tracing below?


a. HCM  MAYO
b. Severe AS
c. Severe AR
d. Severe LV systolic dysfunction

38. The patient in the preceding question is diagnosed with acute pericarditis and
treated successfully with non steroidal anti-inflammatory agents. Over the next
five years, she has recurrent episodes of pericarditis, each treated with
indomethacin. She now presents with the gradual onset of lower extremity edema
over six months. She has vague symptoms of exertion dyspnea and abdominal
bloating but doesn’t feel acutely ill. When you see her in the office, she has obvious
edema and distended neck veins although her lung fields are clear. Her voltage on
ECG is not significantly different from her baseline, and an office echocardiogram
suggests normal left ventricular function and no significant effusion.
Which of the following findings would you expect to see during right heart
catheterization?
a. Increased ventricular fling
b. Right atrial pressure of 3 mmHg
c. Right atrial pressure tracing with a steep Y descent  BRAUNWALD
d. Increased cardiac output
e. Left ventricular end diastolic pressure of 25 mm Hg and a right ventricular
end diastolic pressure of 10 mm Hg

39. A 60 year-old-man with AF was started on warfarin 8 months ago. He present to


the emergency room with severe bleeding. Blood work levels that INR of 4.9.
Hemoglobin 12 mg/dl and platelet count 200.000. You decide to immediately
reverse his warfarin with?
a. Cryoprecipitate
b. Dyalisis
c. Vitamin K
d. Trombocite transfusion

40. What is the diagnosis from ECG and pressure facing below?
a. Constrictive pericarditis
b. Restrictive cardiomyopathy
c. Ischemic cardiomyopathy
d. Pericardial tamponade  MAYO
41. Regarding calculation of LV mass, Relative Wall Thickness (RWT) ≤ 0.42 is
considered:
a. Either concentric remodeling or concentric hypertrophy if the LV mass index
≤ 95 (female) and ≤ 115 (male)
b. Concentric remodeling if the LV mass index ≤ 115 (female) and ≤ 95 (male)
c. Concentric hypertrophy if the LV mass index ≤ 95 (female) and ≤ 115 (male)
d. Normal if the LV mass index ≤ 115 (female) and ≤ 95 (male)
e. Eccentric hypertrophy if the LV mass index > 95 (female) and > 115
(male) Guideline ACE

42. There is some specific, supportive signs and quantitative parameters in grading of
mitral regurgitation.
Which of following parameter/ criteria is suitable for severe mitral regurgitation:
a. Regurgitant fraction > 50%  Echo manual
b. Regurgitant jet huggingthe entire LA wall (> 75% of LA)
c. Vena contracta> 0.3 cm, with EROA 0.2-0.3 cm2
d. Soft, triangular CW Doppler MR signal
e. Dens, parabolic CW Doppler MR signal

43. A 75-year-old woman is referred urgently to the cardiology clinic. She had a
myocardial infraction 4 years earlier, percutaneous coronary intervention with a
stent for angina 12 months earlier and has had two blackouts in the last month, 3
weeks apart. She tells you that in one occasion she was gardening and trying to lift
a heavy plant pot. She had no warning andsuddenly found herself on the ground.
She was alert on recovery. There was no seizure-like activity. She does have
exertion breathlessness although she can manage 400m on the flat and a single
flight of stairs. She has not had angina since her coronary stent 12 months earlier.
Occasionally, she feels light-headed if she stand up too quickly. She is currently
taking aspirin, a beta blocker, an ACE inhibitor, a loop diuretics and a statin. Her
physical examination reveals blood pressure 130/55 mmHg, resting pulse 55 bpm,
regular, normal volume. The JVP is raised by 2 cm, her apex beat is displaced to the
lateral clavicular line, sixth intercostals space and there is a systolic murmur heard
all over the precordium and in the carotids. The lung field are clear and there is
mild pitting edema at the level of her shins.
What is the most appropriate immediate investigation should be performed?
a. Echocardiogram
b. 12-lead ECG  UNCOVER case 16
c. Carotid sinus massage
d. Chest X-ray
e. Lying and standing blood pressure

44. A 70-year-old woman who is new to your practice is evaluated because of


progressive exertional dyspnea for 3 months. She reports recent onset of
orthopnea and paroxysmal nocturnal dyspnea. On physical examination, blood
pressure is 150/80 mmHg, and heart rate is 95/ min. She has normal sinus
rhythm, an S3 and an S4, no murmur, fine bibasilar crackles, and 1 + edema.
Which of the following is the most important test in the evaluation of this patient
to guide initial therapy?
a. Chest radiography
b. Coronary angiography
c. Echocardiography
d. Measurement of the plasma B-type natriuretic peptide
e. Electrocardiography

45. A 70-year-old woman is admitted via ambulance to emergency room. She is very
dyspnea and unable to give history. Medications found with her are as follows :
furosemide 80 mg po od, atenolol 50 mg po od, warfarin, digoxin 0.125 mg po od.
Routine observations are as follows: temperature 37oC, pulse 130 bpm, irregularly
irregular, blood pressure 190/100 mmHg, respiratory rate 40 breath/min, O2
saturations are 88% on 10 L/min O2 through rebreathe mask. Examination
reveals central cyanosis and cool peripheries. Auscultation of the chest reveals
widespread inspiratory crepitation; pulses are absent below the femoral arteries
in both legs. The following investigations are available: arterial blood gases, pH
7.12; pO2 5.8kPa; pCO2 3.2 kPa; bicarbonate 6.0 mmol/L. Routine electrolytes;
sodium 130mmol/L; potassium 5.5 mmol/L; creatinine 3.3 mg/dl; glucose 108
mg/dl. Her CXR shows cardiomegaly, air showing and Kerley B-lines. EKG
recording demonstratewidespread ST depression and ST elevation in lead
aVR.Additional findings on cardiovascular examination are of a prominent apex
beat, which is displace to mid-axillary line. There is a soft systolic murmur present
throughout the precordium and a gallop rhythm.What is the most likely causes in
this woman after evaluate all of the history and clinical examination?
a. Acute on chronic renal failure
b. Dilated cardiomyopathy
c. Critical myocardial ischemia  UNCOVER
d. Flash pulmonary edema associated with renal failure
e. Acute myocardial infarction

46. A 70 year old woman present to your office with dyspnea and peripheral edema.
On examination, her BP is 180/70 mmHg and her pulse is 100 bpm. She has
elevated jugular venous pressure, peripheral edema of the ankles and a fourth
heart sound. All of the following would be reasonable to obtain in the near future
EXCEPT
a. Serum electrolytes, complete blood count and a urinalysis
b. Six minute walking test
c. Echocardiogram
d. ECG and chest x-ray
e. Holter Monitoring  MKSAP 13

47. A 70 year-old man who has cough, shortness of breath, chronic cardiomyopathy,
and an ejection fraction 0f 30% is hospitalized with an exacerbation of pulmonary
edema. He takes digoxin, an ACE inhibitor, and a diuretic. On physical examination,
blood pressure is 110/70 mmHg, and heart rate is 70x/min and regular. He has
bibasilar crackles, an S3 and 1 + edema of the leg.
Which of the following is the most appropriate next step in the management of this
patient?
a. Start treatment with a ?-blocker
b. Discontinue the ACE inhibitor and start treatment with an angiotensin
receptor blocker
c. Increase dieresis  MKSAP 13
d. Start long-term treatment with dobutamine
e. Increase the dosage of digoxin

48. Pseudonormalization of filling pattern is seen in :


a. Stage II LV diastolic dysfunction
b. Stage III LV diastolic dysfunction
c. Stage I LV diastolic dysfunction
d. Stage IV LV diastolic dysfunction

49. A 75-year old woman is referred urgently to the cardiology clinic. she had a
myocardial infarction 4 years earlier, percutaneous coronary intervention with a
stent for angina 12 months earlier and has had two blackouts in the last month, 3
weeks apart. She tells you that in one occasion she was gardening and trying to lift
a heavy plant pot. She has no warning and suddenly found herself on the ground.
She was alert on recovery. There was no seizure-like activity. She does have
exertional breathlessness although she can manage 400 m on the flat and a single
flight of stairs. She has not had angina since her coronary stent 12 months earlier.
Occasionally she feels light-headed if she stand up too quickly. She is currently
taking aspirin, a beta-blocker, an ACE inhibitor, a loop diuretics and a statin. Her
physical examination reveals blood pressure 13-/55 mmHg, resting pulse 55 bpm,
reguler, normal volume. The JVP is raised by 2 cm, her apex beat is displaced to the
lateral clavicular line, sixth intercostal space and there is a systolic murmur heard
all over the precordium and in the carotids. The lung field are clear and there is
mild pitting edema at the level of her shins.
If you choose ICD implantation as a class I indication in patient like this, the reason
you choose this one are based on all of the following EXCEPT?
a. Ejection fraction<35%
b. After has had a non-hemorrhagic stroke with atrial fibrillation  UNCOVER
case 16
c. Ischemic heart disease for arrhythmia prevention
d. Syncope of unknown origin
e. Sustain monomorphic ventricular tachycardia
50. A 35-year old man who underwent closure of an atrial septal defect at age 5 years
was asymptomatic and physically active until 3 months ago, when he began to
have exertional dyspnea and fatigue. He smokes one pack of cigarettes daily and
drinks a six-pack of beer daily. He is a bricklayer and had to stop working for the
last 2 weeks. He takes no medications. On physical examination, blood pressure is
105/80 mmHg, and heart rate is 100/min, with an occasional extrasystole. Jugular
venous pressure is 11 cmH2O. the point of maximal impulse is displaced. The
patient has a soft S1, a split S2, and a grade 2/6 apical holosystolic murmur. The
abdomen is distended, the liver is palpable 1 cm below the right costal margin, and
2 + pedal edema is noted.
Laboratory tests show a total serum cholesterol of 180 mg/dL, serum thyroid-
stimulating hormone of 2.5 ?U/mL, blood urea nitrogen of 32 mg/dL, serum
creatinine of 1.3 mg/dL, alkaline phosphatase of 220 U/L, aspartate
aminotransferase of 60U/L, alanine aminotransferase of 75 U/L, and serum
bilirubin of 1.2 mg/dL. Electrocardiogram show non-diagnostic ST changes, with
occasional premature ventricular contractions.Gated single-photon emission
tomography scan shows an ejection fraction of 34%, with global hyperkinesia.
Which of the following is the most likely cause of this patients heart failure?
a. Late heart failure as a result of repair of an atrial septal defect
b. Familial dilated cardiomyopathy
c. Coronary artery disease
d. Alcohol consumption - MKSAP 13 no 103
e. Atrial septal defect patch dehiscence

51. A 75-year-old woman is referred urgently to the cardiology clinic. She had a
myocardial infarction 4 years earlier, percutaneous coronary intervention with a
stent for angina 12 months earlier and has had two blackouts in the last month, 3
weeks apart. She tells you that in one occasion she was gardening and trying to lift
a heavy plant pot. She had no warning and suddenly found herself on the ground.
She was alert on recovery. There was no seizure-like activity. She does have
exertional breathlessness although she can manage 400 m on the flat and a single
flight of stairs. She has not had angina since her coronary stent 12 months earlier.
Occasionally she feels light-headed if she stand up too quickly. She is currently
taking aspirin, a beta blocker, an ACE inhibitor, a loop diuretic and a statin. Her
physical examination reveals blood pressure 130/50 mmHg, resting pulse 55 bpm,
regular, normal volume. The JVP is raised by 2 cm, her apex best is displaced to the
lateral clavicular line, sixth intercostal space and there is a systolic murmur heard
all over the precordium and in the carotids. The lung field are clear and there is
mild pitting edema at the level of her shins.
All of the following would be the cause of blackout in this patient,EXCEPT?
a. A bradycardia caused by Stokes-Adam attack
b. A tachycardia caused by ventricular tachycardia
c. Transient ischemic attack  UNCOVER Case 16
d. Orthostatic hypotension
e. Severe aortic stenosis

52. A 70-year old woman is admitted via ambulance to emergency room. She is very
dyspneic and unable to give history. Medications found with her as follows
:furosemide 80 mg po od, atenolol 50 mg po od, warfarin, digoxin 0.125 mg po od.
Routine observation are as follows: temperature 37 C, pulse 130 bpm, irregularly
irregular, blood pressure 190/100 mmHg, respiratory rate 40 breath/min, O2
saturation are 88 % on 10 L/min O2 through rebreathe mask. Examination reveals
central cyanosis and cool peripheries. Auscultation of the chest reveals widespread
inspiratory crepitation; pulses are absent below the femoral arteries in both legs.
The following investigations are available: arterial blood gases, pH 7.12;pO2
5.8kPa; pCO2 3.2kPa; bicarbonat 6.mmol/L. routine electrolytes; sodium 130
mmol/L; potassium 5.5mmol/L; creatinine 300 umol/L;glucose 6.0 mmol/L. her
CXR shows cardiomegaly, air shadowing and Kerley B lines. EKG recording
demonstrate widespread ST depression and ST elevation in lead aVR.
What is the probable working diagnosis:
a. Pulmonary emphysema and cor pulmonary
b. Hypertensive heart failure
c. Pulmonary edema acidosis  UNCOVER hal 90
d. Bilateral pleural effusion
e. Pulmonary emboli and severe pneumonia
53. A 70-year old woman is admitted via ambulance to emergency room. She is very
dyspneic and unable to give history. Medications found with her as follows :
furosemide 80 mg po od, atenolol 50 mg po od, warfarin, digoxin 0.125 mg po od.
Routine observation are as follows: temperature 37 C, pulse 130 bpm, irregularly
irregular, blood pressure 190/100 mmHg, respiratory rate 40 breath/min, O2
saturation are 88 % on 10 L/min O2 through rebreathe mask. Examination reveals
central cyanosis and cool peripheries. Auscultation of the chest reveals widespread
inspiratory crepitation; pulses are absent below the femoral arteries in both legs.
The following investigations are available: arterial blood gases, pH 7.12;pO2
5.8kPa; pCO2 3.2kPa; bicarbonat 6.mmol/L. routine electrolytes; sodium 130
mmol/L; potassium 5.5mmol/L; creatinine 300 umol/L;glucose 6.0 mmol/L. her
CXR shows cardiomegaly, air shadowing and Kerley B lines. EKG recording
demonstrate widespread ST depression and ST elevation in lead aVR.
All of the following are the findings ont the arterial blood gasses, EXCEPT :
a. Profound lactic acidosis
b. Metabolic acidosis related to renal failure
c. Severe hyperkalemia due to renal dysfunction  UNDERCOVER
d. Type 1 respiratory failure
e. A ventilation perfusion mismatch

54. A 56-year-old woman who is new to your practice is evaluated for recent
exacerbation of dyspnea and fatigue. She has idiopathic dilated cardiomyopathy
and receives a stable heart failure regimen, including lisinopril 20 mg/day; digoxin
0.125 mg/d; furosemide 40 mg/d; and metoprolol XL 50 mg/d. she also takes
alendronate, hormone replacement therapy, glipizide, folic acid and ibuprofen
because of rheumatoid arthritis. Thyroid hormone suppressive therapy with
thyroxin was initiated because of the finding of an elevated serum thyroid serum
level 4 months earlier. The thyroid-stimulating hormone level returned to normal
after therapy.
On physical examination, blood pressure is 110/72 mmHg, and heart rate is
82/min. jugular venous pressure is estimated at 10 cm H20. The lungs are clear.
Cardiac examination shows an S3 gallop and 2+ pitting edema.
Which of the following is most likely to be causing the exacerbation of congestive
heart failure in this patient?
a. Alendronate
b. Thyroxin
c. Glipizide
d. Estrogen
e. Ibuprofen  MKSAP 13

55. An 70-year woman is admitted via ambulance to emergency room. She is very
dyspnea and unable to give history. Medication found with her are as follows:
furosemide 80 mg pood, atenolol 50 mg po od, warfarin, digoxin 0.125 mg po od.
Routine observations are as follows: temperature 37 C, pulse 130 bpm, irregularly
irregular, blood pressure 190/100 mmHg, respiratory rate 40 beat/min, O2
saturation are 88% on 10 L/min O2 through rebreathe mask. Examination reveals
central cyanosis and cool peripheries. Auscultation of the chest reveals widespread
inspiratory crepitation; pulses are absent below the bicarbonate 6 mmol/L.
routine electrolytes: sodium 130 mmol/L; potassium 5.5 mmol/L;creatinine 300
umol/L; glucose 6 mmol/L. Her CXR shows cardiomegaly, air shadowing and
Kerley B-lines, EKG recording demonstrate widespread ST depression and ST
elevation in lead aVR.
Then patients is given 40 mg of furosemide intravenously and commenced on
intravenous nitrate. One hour after commencing therapy she remains acutely
unwell. O2 saturation are 86% on 10 L O2 via rebreathe mask, blood pressure is
now 140/70 mmHg and the respiratory rate is 35 breaths/min. there has been no
urine output.
What other intervention would be the most valuable you consider?
a. Hemofiltration
b. Continuous positive airways pressure  UNDERCOVER
c. High-dose intravenous diuretics
d. Administration of cardiac glycoside
e. Intra aorticballooncounter pulsation
56. A 55-year-old woman is hospitalized because of orthopnea and paroxysmal
nocturnal dyspnea. Echocardiography show an ejection fraction 20% and a
diagnosis of dilated cardiomyopathy is made. Diuretic treatment is initiated,
beginning with furosemide administered intravenously and followed by
furosemide, 40 mg twice a day orally. Treatment with an CAE inhibitor is also
initiated. On admission, the blood urea nitrogen level is 25 mg/dL and serum
creatinine level is 1.6 mg/dL. After the dose of captopril is increased from 6.25 mg
three times a day to 12.5 mg three times a day on hospital day 2, the blood urea
nitrogen level is 42 mg/dL, creatinine level is 2.0 mg/dL, and potassium level is 4.6
meq/L. Net diuresis in the hospital is 2.4 L over 36 hours. On physical examination,
blood pressure is 105/85 mm Hg, and heart rate is 88/min. the crackles have
resolved, and jugular venous pressure is normal. Trace pedal edema is noted.
Which of the following is the most appropriate next step in the management of this
patient?
a. Start treatment with low dose dobutamine
b. Discontinue captopril
c. Administer a fluid bolus
d. Decrease the dosage of captopril
e. Decrease the dosage of the diuretic  MKSAP 13 no 85

57. A 70-year-old man comes to your office for evaluation before transurethral
resection of the prostate gland. He has a history of hypertension, feels well, leads
an active life and has no symptoms. On physical examination, blood pressure is
140/70 mmHg, and heart rate is 70/min and regular. Jugular venous pressure and
carotid upstroke are normal and an S4 is heard. The patient has no S3, no murmur
and no peripheral edema. A preoperative chest radiograph shows cardiomegaly.
Echocardiogram performed to evaluate cardiomegaly shows left ventricular
hypertrophy, with global left ventricular dysfunction and an ejection fraction of
33%. No valvular abnormalities are noted. ACE inhibitor therapyis initiated.
Which of the following medication should also be given to this patient?
a. Diuretic
b. Digoxin
c. ?-blocker  MKSAP 13 no 88
d. Angiotensin receptor blocker
e. Spironolactone
58. A 35-year-old male presents to ED with 3 weeks of edema and progressive
shortness of breath. BP 110/80 mmHg, HR 110 bpm, O2 sat 88%. Bedside
echocardiogram shows LVEF 25%. Which of the following therapies is most likely
to acute worsen the patients status?
a. Nitoglycerine 10 mcg/min IV gtt
b. Captopril 12.5 mg PO
c. Carvedilol 3.125 mg PO  Hasil diskusi, Sumber soal belum ditemukan
d. Furosemide 40 mg IV

59. A 74-year-old woman is admitted with 3 hours of crushing substernal chest pain.
She has a history of left carotid occlusion with hemiparesis occurring 3 months
ago. She also has a history of mild hypertension, hyperlipidemia, and diabetes
mellitus complicated by neuropathy and retinopathy. Her medications include
warfarin, 5 mg/d; atenolol 25 mg/d; and pravastatin 20 mg/d. in the emergency
department, she has a ventricular fibrillation arrest. She is successfully
cardioverted to sinus rhythm after receiving 2 minutes of cardiopulmonary
resuscitation. Her electrocardiogram shows sinus rhythm with 3 mm ST segment
elevation in V2-6. The result of initial laboratory tests are within normal limits,
except for an elevated prothrombin time with an INR of 1.8.
Which of the following represent an absolute contraindication to the use of a
thrombolytic agent in this patient?
a. Left carotid occlusion with hemiparesis 3 months ago MKSAP 13 no 97
b. Patient age > 70 years
c. Patient on warfarin, with an INR of 1.8
d. Cardiopulmonary resuscitation for 2 minutes
e. Diabetic retinopathy

60. A 73-year-old woman comes to the emergency department because of a 2-hour


history of severe precordial chest pain and dyspnea. She has no significant past
medical history, and is talking no medication. An electrocardiogram shows 2 mm
ST segment elevation in leads V3-6. The patient is promptly treated with chewable
aspirin, intravenous heparin and fibrinolysis with front-loaded tissue plasminogen
activator, following which her symptoms resolve and the ST segments return to
normal. She is admitted to the coronary care unit.
On physical examination, her pulse rate is 78/min and regular, and blood pressure
is 120/76 mmHg. There are bibasilar crackles, and an S4 is present at the apex. She
is prescribed aspirin,325 mg daily, atenolol 50 mg daily, isosorbidemononitrate,
30 md daily, furosemide 40 mg orally and intravenous heparin with a goal of
achieving an activated partial thromboplastin time of 55 to 80 seconds. Laboratory
studies 12 hours after admission show a markedly elevated serum troponin I level
(50 ng/mL). a repeat electrocardiogram shows evidence of an evolving
anterolateral myocardial infarction with Q waves and ST segment depression in
leads V3-6. Telemetry over the first 12 hours shows four episodes of
asymptomatic, monomorphic non-sustained ventricular tachycardia, each episode
lasting 6 to 15 beats. A transthoracic echocardiogram shows a large anterior wall
motion abnormality and an ejection fraction of 35%.
Which of the following should you also prescribe?
a. Amiodarone
b. Sotalol
c. Captopril  MKSAP 13 no 86
d. Amlodipine
e. Diltiazem

61. A 74-year-old man is resuscitated after an out-of-hospital cardiac arrest. He does


not have a myocardial infarction, but had two earlier myocardial infarction. His left
ventricular ejection fraction is 32%. An adenosine thalium scan shows anterior
and inferior scarring, but no ischemia. Telemetry shows that he has three to four
episodes of non-sustained ventricular tachycardia daily. He has moderately severe
post-anoxic encephalopathy. On medical therapy, he has no signs of heart failure.
His laboratory findings are normal. His family would like reasonable steps to be
taken, but do not wish him to undergo surgery, including placement of an
implantable cardioverter defibrillator.
Which of the following drugs would be most effective in this patient?
a. Amiodarone
b. Diltiazem
c. Sotalol
d. Carvedilol  MKSAP 13 no 87
e. Procainamide

62. A 25-years-old pregnant woman (G1P0A0, gestational age 32 weeks) was


admitted to the hospital with main complaint shortness of breath and orthopnea.
She never complained this symptoms before. Patient also complain hemoptysis.
On physical examination, Blood pressure 100/60 mmHg, Heart rate was 132
x/minute irregularly irregular, Respiratory rate 28 breaths/minute. Patient had
distended jugular vein and left parasternal heave. Cardiac auscultation revealed
low pitched rumbling mid diastolic murmur.Chest X ray revealed congested upper
lobe vein and Kerley B lines.
Which of the following is the most appropriate anti thrombotic drugs for this
patients?
a. Unfractioned heparin
b. Fondaparinux
c. Warfarin
d. Dabigatran
e. Low molecular weight heparin

63. A 38-year-old man came to emergency department with chief complaint shortness
of breath, non-radiating chest tightness and 2 syncopal episodes. The symptom has
been felt since 1 month before admission, but it was going worse in the recent 1
week. From the anamnesis, one week before admission the patient felt pain and
swelling on upper right leg, after he drove a car for two and a half hours. The leg
was then being massaged and the symptoms were going worse and he started
feeling shortness of breath. From the medical history, the patient was obese (Body
Mass Index 31 kg/m2), and has history of smoking for 19 years. The patient was a
frequent distance traveler (average duration 4 to 6 hours for about 12
times/month). Patient has no history of hypertension and diabetes. On admission,
his blood pressure was 90/60 mmHg, pulse 120 times/minute and regular,
respiratory rate 26 breaths/minute, temperature 36oC, and oxygen saturation
89%. Other physical examinations were unremarkable.
Which of the following is the most likely diagnosis for this patient?
a. Aortic Dissection
b. Cardiac Tamponade
c. Acute Myocardial Infarction
d. Pulmonary Embolism  Hasil diskusi, sumber soal belum ditemukan
e. Acute Lung Oedema

64. A 38-year-old man came to emergency department with chief complaint shortness
of breath, non-radiating chest tightness and 2 sincopal episodes. The symptom has
been felt since 1 month before admission, but it was going worse in the recent 1
week. From the anamnesis, one week before admission the patient felt pain and
swelling on upper right leg, after he drove a car for two and a half hours. The leg
was then being massaged and the symptoms were going worse and he started
feeling shortness of breath. From the medical history, the patient was obese (Body
Mass Index 31 kg/m2), and has history of smoking for 19 years. The patient was a
frequent distance traveler (average duration 4 to 6 hours for about 12
times/month). Patient has no history of hypertension and diabetes. On admission,
his blood pressure was 90/60 mmHg, pulse 120 times/minute and regular,
respiratory rate 26 breaths/minute, temperature 36oC, and oxygen saturation
89%. Other physical examinations were unremarkable.
Which is the golden diagnostic tool for the above patient?
a. Coronary Angiography
b. Cardiac Marker
c. Chest CT Angiography  UNCOVER
d. Echocardiography
e. Electrocardiography

65. A 38-year-old man came to emergency department with chief complaint shortness
of breath, non-radiating chest tightness and 2 sincopal episodes. The symptom has
been felt since 1 month before admission, but it was going worse in the recent 1
week. From the anamnesis, one week before admission the patient felt pain and
swelling on upper right leg, after he drove a car for two and a half hours. The leg
was then being massaged and the symptoms were going worse and he started
feeling shortness of breath. From the medical history, the patient was obese (Body
Mass Index 31 kg/m2), and has history of smoking for 19 years. The patient was a
frequent distance traveler (average duration 4 to 6 hours for about 12
times/month). Patient has no history of hypertension and diabetes. On admission,
his blood pressure was 90/60 mmHg, pulse 120 times/minute and regular,
respiratory rate 26 breaths/minute, temperature 36oC, and oxygen saturation
89%. Other physical examinations were unremarkable.
Below is ECG findings that can be found in the above patient, EXCEPT :
a. Sinus Tachycardia
b. Atrial arrhythmia with P pulmonale
c. Left Bundle Branch Block  UNCOVER
d. ST elevation with Right Bundle Branch Block in V1-V2
e. Inverted T wave in anterior lead

66. 68-year-old woman presents to your office for initial evaluation. She has had
progressive dyspnea over the past 2 years. She has long-standing hypertension
and reports tobacco use (50 pack-years). She has been treated with Bosentan for
idiopathic PAH (iPAH). Currently, she is assessed as having World Health
Organization (WHO) functional Class III limitations.Evaluation revels the following
:
Echocardiogram : ejection fraction 66%, grade 3 diastolic abnormality, moderate
left ventricular hypertrophy, no significant valvular disease.
Right heart catheterization : right atrial pressure 8 mmHg, right ventricular
pressure 45/20 mmHg, pulmonary artery pressure 50/24 mmHg, mean
pulmonary artery pressure 33 mmHg, pulmonary capillary wedge pressure 25
mmHg, and cardiac output 5,5 L/min.
Which of the following treatments would you recommend at this time?
a. Phophodiesterase-5-inhibitor
b. Diuretics, salt restriction, and blood pressure control
c. Increase Bosentan dose
d. Cardiac resynchronization therapy
67. A 45-year old business executive presents to A&E with a 2-hour history of central
crushing chest pain and breathlessness. He is a non-smoker, previously very fit
and well and attends a gym four times a week. There is no family history of
ischemic heart disease. His cholesterol measured at an insurance medical was 3,3
mmol/L. his observations on admission are as follows; pulse 105 bpm; blood
pressure 80/50 mmHg; O2 saturations 90% on room air. He is apyrexial. An ECG is
performed and shown sinus tachycardia of 105 bpm. Right-axis deviation and non-
specific T-wave inversion in leads III, aVF, V2-V4.
Based on the ECG findings, what is your working diagnosis?
a. Chronic corpulmonale
b. Pulmonary embolism  UNCOVER
c. Paroxysmal rapid atrial dysrhythmias
d. Right ventricular dysplasia
e. Primary pulmonary hypertension

68. Which of the following tests would be appropriate to confirm a diagnosis of PAH in
a patient with elevated right ventricular systolic pressure demonstrated by
echocardiography?
a. Liver function tests
b. Antinuclear antibody
c. Right Heart Catheterization  UNCOVER Case 16
d. Transesophageal echocardiography

69. You are asked to see a 50 year old female dietitian in consultation for HTN. She
was found to have an elevated BP on medical check-up 4 years ago. She followed
her physician’s recommendations and uses only sodium substitutes, limits alcohol
consumption, and exercises. She adopted a vegetarian lifestyle. Despite these
measures, her BP remained above normal and health care provider prescribed
several medications regimens. However, her BP could not be maintained at
<160/90 mmHg. Her medications include: Metoprolol 25 mg twice daily,
Lisinopril 20 mg twice daily, Amlodipine 10 mg daily.
Your examination detects the following : BP : 188/100 mmHg (seated), 190/100
mmHg (standing); HR : 70 bpm, siting, 80 bpm standing, Normal
funduscopicexamination, Normal peripheral pulses and no abdominal bruits,
Normal cardiopulmonary examination.
Of the following statements regarding the clinical presentation, which is correct?
a. The HTN is not “resistant” because the patient is not taking appropriate
medications at their maximum doses  MAYO
b. The next step should be US assessment of renal arterial flow
c. The absence of an abdominal bruit excludes renovascular HTN as the
underlying diagnosis
d. The BP response to postural change suggests a state of low volume-high
resistance HTN

70. A 38-year-old man came to emergency department with chief complaint shortness
of breath, non-radiating chest tightness and 2 sincopal episodes. The symptom has
been felt since 1 month before admission, but it was going worse in the recent 1
week. From the anamnesis, one week before admission the patient felt pain and
swelling on upper right leg, after he drove a car for two and a half hours. The leg
was then being massaged and the symptoms were going worse and he started
feeling shortness of breath. From the medical history, the patient was obese (Body
Mass Index 31 kg/m2), and has history of smoking for 19 years. The patient was a
frequent distance traveler (average duration 4 to 6 hours for about 12
times/month). Patient has no history of hypertension and diabetes. On admission,
his blood pressure was 90/60 mmHg, pulse 120 times/minute and regular,
respiratory rate 26 breaths/minute, temperature 36oC, and oxygen saturation
89%. Other physical examinations were unremarkable.
What is the most important therapy for above patients?
a. Pericardiocentesis
b. Fibrinolysis  UNCOVER
c. Primary Percutaneous Intervention
d. Urgent surgery
e. Low Molecular Weight Heparin
71. You are asked to see a 50 year old female dietitian in consultation for HTN. She
was found to have an elevated BP on medical check-up 4 years ago. She followed
her physician’s recommendations and uses only sodium substitutes, limits alcohol
consumption, and exercises. She adopted a vegetarian lifestyle. Despite these
measures, her BP remained above normal and health care provider prescribed
several medications regimens. However, her BP could not be maintained at
<160/90 mmHg. Her medications include: Metoprolol 25 mg twice daily,
Lisinopril 20 mg twice daily, Amlodipine 10 mg daily.
Your examination detects the following : BP : 188/100 mmHg (seated), 190/100
mmHg (standing); HR : 70 bpm, siting, 80 bpm standing, Normal funduscopic
examination, Normal peripheral pulses and no abdominal bruits, Normal
cardiopulmonary examination.
Serum aldosterone concentration in this patient was 2 ng/dL (Normal: 1-21
ng/dL).
Which of the following substances might be playing a role in this patient’s HTN?
a. Alcohol
b. Angiotensin II  MAYO
c. Diuretic
d. Premarin

72. Which of the following is the most common presenting symptom in patients with
PAH?
a. Presyncope
b. Dyspnea
c. Fatigue
d. Chest pain

73. A 38-year-old man came to emergency department with chief complaint shortness
of breath, non-radiating chest tightness and 2 sincopal episodes. The symptom has
been felt since 1 month before admission, but it was going worse in the recent 1
week. From the anamnesis, one week before admission the patient felt pain and
swelling on upper right leg, after he drove a car for two and a half hours. The leg
was then being massaged and the symptoms were going worse and he started
feeling shortness of breath. From the medical history, the patient was obese (Body
Mass Index 31 kg/m2), and has history of smoking for 19 years. The patient was a
frequent distance traveler (average duration 4 to 6 hours for about 12
times/month). Patient has no history of hypertension and diabetes. On admission,
his blood pressure was 90/60 mmHg, pulse 120 times/minute and regular,
respiratory rate 26 breaths/minute, temperature 36oC, and oxygen saturation
89%. Other physical examinations were unremarkable.
Which of the following echocardiographic finding that can be found in above
patients?
a. Right ventricular free wall hypokinesis in the presence of normal right
ventricular apical.
b. Acute mitral regurgitation
c. Hypokinesis of the anterior and inferior wall
d. Visualization of true lumen and false lumen
e. RV acceleration time >60 ms in the presence of tricuspid insufficiency
pressure gradient >60 mmHg  UNCOVER, diskusi

74. A 20-year-old woman is in the first trimester of her first pregnancy is referred to
you because of progressive dyspnea. An echocardiogram shows findings consistent
with severe pulmonary hypertension. The right ventricular systolic pressure is 90
mmHg, and systemic blood pressure is 110 mmHg. Further studies show no
evidence of pulmonary embolism. Transesophageal echocardiogram excludes an
intracardiac shunt.
Which of the following is associated with the lowest risk of maternal mortality?
a. Initiation of bosentan therapy
b. Initiation of prostacyclin therapy
c. Termination of pregnancy  MKSAP no 98
d. Initiation of ACE inhibitor therapy
e. Initiation of anticoagulation therapy
75. Which of the following patients with an LVEF of 25% would be most appropriate
to refer for a right heart catheterization?
a. An 85-year-old man with a 30-year history of diabetes previously received
laser photocoagulation therapy of diabetic retinopathy, and has ongoing leg
pain from neuropathy. He now presents with shortness of breath, has a blood
pressure of 200/100 mmHg, anasarca, and a creatinine of 10.
b. A 60-year-old man with an LVEF of 25% is admitted with shortness of breath
and is found to have blood pressure 130/80 mmHg, pulse 90 bpm, and JVP of
14 cm with 2+ leg edema and warm extremities, with creatinine 1.2.
c. A 30-year-old woman is admitted to the hospital with shortness of breath and
blood pressure of 90/70 mmHg. Following 2 days of intravenous diuretics,
her examination is notable for JVP of 16 cm and 2+ leg edema, with a systolic
blood pressure of 78 mmHg. Her creatinine has increase from 1.6 to 2.5, with
diuresis.
d. A 20-year-old man is referred for cardiac transplantation due to his low LVEF
despite 6 months of beta-blocker therapy. He reports mild fatigue while
playing racquetball and his examination shows blood pressure 120/80
mmHg, pulse 72 bpm, JVP <8 with no hepatojugular reflux, and there is no
S3. Extremities are warm to touch, with no edema  Hasil diskusi
e. A 40-year-old woman with asthma and heart failure is admitted to the
hospital. Her examination is notable for blood pressure 150/80 mmHg; pulse
110 bpm and regular; lungs with diffuse expiratory wheezes; JVP <8 cm with
no hepatojugular reflux; cardiac auscultation with S12 and no S3; legs with
no edema and are warm to touch.

76. A 45 year old business executive presents to A&E with a 2-hour history of central
crushing chest pain and breathlessness. He is a non-smoker, previously very-fit
and well and attends a gym four times a week. There is no family history of
ischemic heart disease. His cholesterol measured at an insurance medical was 127
mg/L. His observations on admission are as follows; pulse 105 bpm; blood
pressure 80/50 mmHg; O2 saturations 90% on room air. He is apyrexial. An ECG is
performed and shown sinus tachycardia of 105 bpm. Right-axis deviation and non-
specific T-wave inversion in Leads III,aVF,V2-V4.
For immediate management, you can perform all of the following, EXCEPT?
a. Opiates should be avoided if possible
b. Airway breathing and circulation is required
c. The patients should be nursed lying flat or with a head-down tilt 
UNCOVERED
d. High Flow Oxygen
e. Large volume of intravenous fluids

77. 68 year old woman presents to your office for initial evaluation. She has had
progressive dyspnea over the past 2 years. She has long-standing hypertension
and reports tobacco use (50 pack-years). She has been treated with bosentan for
idiopathic PAH(iPAH). Currently, she is assessed as having World Health
Organization (WHO) functional class III limitations. Evaluation reveals the
following:
Echocardiogram: ejection fraction 66%, grade 3 diastolic abnormality, moderate
left ventricular hypertrophy, no significant valvular disease.
Right Heart Catheterization: right atrial pressure 8 mmHg, right ventricular
pressure 45/20 mmHg, pulmonary artery pressure 50/24 mmHg, mean
pulmonary artery pressure 33 mmHg, pulmonary capillary wedge pressure 25
mmHg, and cardiac output 5/5 L/min.
Which of the following scenarios may occur upon acute vasodilator challenge
(pulmonary hypertension reactivity test) in this patient?
a. The patient may not respond and can then be considered a candidate for
chronic calcium channel blocker therapy
b. Altered mental status
c. Acute Renal Failure
d. Acute life-threatening pulmonary edema in the setting of a markedly elevated
pulmonary capillary wedge pressure

78. A 45 year old business executive presents to A&E with a 2-hour history of central
crushing chest pain and breathlessness. He is a non-smoker, previously very fit
and well and attends a gym four times a week. There is no family history of
ischemic heart disease. His cholesterol measured at an insurance medical was 3.3
mmol/L. His observations on admission are as follows; pulse 105 bpm; blood
pressure 80/50 mm Hg; O2 saturations 90% on room air. He is apyrexial. An ECG
is performed and shown sinus tachycardia of 105 bpm. Right-axis deviation and
non-specific T wave inversion in leads III,aVF, V2-V4.
All of the following aspects of the history and examination would help you to
establish diagnosis, EXCEPT?
a. Attenuated pulmonary second sound
b. Absence of exertional angina
c. Radiographic evidence of pulmonary edema  UNCOVER
d. Family history of thromboembolic disease
e. The patient may have recently been on a long-haul flight

79. A 45 year old business executive presents to A&E with a 2-hour history of central
crushing chest pain and breathlessness. He is a non-smoker, previously very fit
and well and attends a gym four times a week. There is no family history of
ischemic heart disease. His cholesterol measured at an insurance medical was 3.3
mmol/L. His observations on admission are as follows; pulse 105 bpm; blood
pressure 80/50 mm Hg; O2 saturations 90% on room air. He is apyrexial. An ECG
is performed and shown sinus tachycardia of 105 bpm. Right-axis deviation and
non-specific T wave inversion in leads III,aVF, V2-V4.
The patient respons to the intravenous fluid replacement (blood pressure 100/70
mmHg, pulse 100 bpm) but remains hypoxic (O2 saturation 90%).
What further imaging may be helpful at this point?
a. CT pulmonary angiogram has a moderate sensitivity for acute corpulmonale
b. Transthoracic echocardiogram can be very useful diagnostically 
UNCOVER
c. Chest X-ray is enough to confirm pulmonary embolic disease
d. Perfusion scan can help to exclude pulmonary hypertension
e. Ventilation perfusion scan can identifying chronic corpulmonale
80. You are asked to see a 50 year old female dietitian in consultation for HTN. She
was found to have an elevated BP on a medical check-up 4 years ago. She followed
her physician’s recommendations and uses only sodium substitutes, limits alcohol
consumption, and exercises. She adopted a vegetarian lifestyle. Despite these
measures, her BP remained above normal and her health care provider prescribed
several medication regimens. However, her BP could not be maintained at
<160/90 mmHg. Her medications include: Metoprolol 25 mg twice/daily,
Lisinopril 20 mg twice/daily, Amlodipine 10 mg/daily. Your examination detects
the following: BP: 188/100 mmHg (seated), 190/100 (standing); HR: 70 bpm,
sitting, 80 bpm standing, Normal funduscopic examination, Normal peripheral
pulses and no abdominal bruits, Normal cardiopulmonary examination.
This patient has continued medical therapy and improves somewhat with addition
of triamterene/HCTZ (37.5/25mg) daily. Her BP is now 160 mmHg systolic.
Laboratory results include:
CBC: normal
Creatinine: 1.9 mg/dL
Sodium: 145 mEq/L
Potassium: 3.5 mEq/L
Uric Acid: 3.0 mg/dL
ECG: LVH by voltage criteria
Chest X-ray: normal
According to this patient, the diagnosis of primary aldosteronism requires each of
the following EXCEPT:
a. Hypertension
b. Hypokalemia (salt replete)
c. Normal renal arteries  MAYO
d. Suppressed plasma renin activity
e. Increased 24-hr urinary aldosterone rate

81. A 45 year old business executive presents to A&E with a 2-hour history of central
crushing chest pain and breathlessness. He is a non-smoker, previously very fit
and well and attends a gym four times a week. There is no family history of
ischemic heart disease. His cholesterol measured at an insurance medical was 3.3
mmol/L. His observations on admission are as follows; pulse 105 bpm; blood
pressure 80/50 mm Hg; O2 saturations 90% on room air. He is apyrexial. An ECG
is performed and shown sinus tachycardia of 105 bpm. Right-axis deviation and
non-specific T wave inversion in leads III,aVF, V2-V4.This patient returned to his
country from a business trip in Australia 2 weeks previously.
On clinical examination, you can find all of the following, EXCEPT?
a. The D-dimer is elevated to the 5 times the normal range
b. Cyanosed and cool peripherally
c. Jugular venous pressure is elevated
d. The troponin I within normal limits (<1 ug/L)  Uncover
e. There is a soft third heart sound over the right sternal border

82. You are asked to see a 50 year old female dietitian in consultation for HTN. She
was found to have an elevated BP on a medical check up 4 years ago. She followed
her physician’s recommendations and uses only sodium substitutes, limits alcohol
consumption, and exercises. She adopted a vegetarian lifestyle. Despite these
measures, her BP remained above normal and her health care provider prescribed
several medication regimens. However, her BP could not be maintained at
<160/90 mmHg. Her medications include: Metoprolol 25 mg twice/daily,
Lisinopril 20 mg twice/daily, Amlodipine 10 mg/daily. Your examination detects
the following: BP: 188/100 mmHg (seated), 190/100 (standing); HR: 70 bpm,
sitting, 80 bpm standing, Normal funduscopic examination, Normal peripheral
pulses and no abdominal bruits, Normal cardiopulmonary examination.
This patient has continued medical therapy and improves somewhat with addition
of triamterene/HCTZ (37.5/25mg) daily. Her BP is now 160 mmHg systolic.
Laboratory results include:
CBC: normal
Creatinine: 1.9 mg/dL
Sodium: 145 mEq/L
Potassium: 3.5 mEq/L
Uric Acid: 3.0 mg/dL
ECG: LVH by voltage criteria
Chest X-ray: normal
The most likely secondary form of HTN in this setting is:
a. Renovascular stenosis
b. Chronic renal failure
c. Pheochromocytoma
d. Primary aldosteronism  MAYO
83. A 35 year old woman who emigrated to the United States is referred to you by her
gynecologist for evaluation of hypertension that was noted 1 week ago, when she
sought an evaluation for infertility. She was first told that she had hypertension at
20 years of age, but did not follow up with a physician until recently. On your
advice, her gynecologist initiated treatment with amlodipine, 5 mg, after obtaining
a blood pressure of 200/100 mmHg. The patient has frequent headaches and also
has cold feet and leg cramping when she walks along distances.
Physical examination shows blood pressure of 160/90 mmHg in the left arm while
sitting and heart rate of 70/min. Jugular venous pressure is normal. Carotid pulses
are brisk bilaterally. Cardiac examination shows a sustained apical impulse. S1 is
normal and S2 is physiologically split. An early systolic ejection sound is noted, and
an early peaking murmur is noted at the second right intercostal space. A short
diastolic murmur is audible along the left sternal border. Lungs are clear to
auscultation. Electrocardiogram shows left ventricular hypertrophy. Findings on
urinalysis are normal.
Which of the following is the most appropriate next step in the evaluation of this
patient?
a. Order a 24-hour urine test for metanephrine and vanillylmandelic acid
b. Obtain a chest radiograph
c. Order an echocardiogram
d. Measure serum thyroid-stimulating hormone
e. Measure the blood pressure in the lower extremities  MKSAP 13 no 89

84. A 68-year-old woman presents to your office for initial evaluation. She has had
progressive dyspnea over the past 2 years. She has long-standing hypertension
and reports tobacco use (50 pack-years). She has been treated with bosentan for
idiopathic PAH(iPAH). Currently, she is assessed as having World Health
Organization (WHO) functional class III limitations. Evaluation reveals the
following:
Echocardiogram: ejection fraction 66%, grade 3 diastolic abnormality, moderate
left ventricular hypertrophy, no significant valvular disease
Right Heart Catheterization: right atrial pressure 8 mmHg, right ventricular
pressure 45/20 mmHg, pulmonary artery pressure 50/24 mmHg, mean
pulmonary artery pressure 33 mmHg, pulmonary capillary wedge pressure 25
mmHg, and cardiac output 5/5 L/min.
Based on your review of her evaluation findings, which of the following represents
the appropriate diagnosis?
a. Pulmonary vein hypertension
b. Chronic Obstructive Pulmonary Disease (COPD)
c. Pulmonary arterial hypertension
d. Viral Cardiomyopathy

85. A 60-years-old man with complaints night cramp, heaviness, pitting edema,
hyperpigmentation and active skin ulcer of left ankle since last year in the bilateral
lower limbs. Varicose veins were seen in the medial of left thigh and calf. Duplex
ultrasound showed severe reflux in the bilateral lower limb but just only
incompetent calf perforator vein with diameter 5 mm in the left lower limb. Duplex
ultrasound showed saphenous junction with diameter 10 mm and great saphenous
diameter 4.5 mm just above knee.
Management therapy is needed in the case above, except?
a. Calf perforator ligation
b. Saphenous junction ligation
c. Varicose phlebectomy
d. Stocking compression  Jawaban angkatan mas hendry
e. Great saphenous ablation

86. Which of the following statements regarding transesophageal findings of aortic


atheroma is not true?
a. Limited data suggest that these patients may benefit from anticoagulation
therapy with warfarin
b. Plaques >2mm in the ascending aorta are associated with increased risk of
stroke  MKSAP 16
c. Mobile components are associated with an increased risk of stroke
d. Plaques >4mm in the ascending aorta are associated with increased risk of
stroke

87. A 29-year-old woman in the 28th week of pregnancy has sudden onset of severe
anterior chest pain radiating to her back. The pain began 2 hours earlier and
increased in severity. It was not associated with shortness of breath, nausea or
vomiting, or diaphoresis. The patient has a history of mitral valve prolapse that
was diagnosed on echocardiogram 10 years earlier. Her only medication is a
prenatal vitamin. Her family history is unremarkable.
On physical examination, blood pressure is 105/78 mmHg, heart rate is 110/min,
and respiration rate is 18/min while the patient is lying still. The patient is
afebrile. Examination of the head, eyes, ears, nose, and throat shows a high, arched
palate. Carotid pulses are normal bilaterally, with no jugular venous distension.
The lungs are clear to auscultation. Cardiac examination shows a non-displaced
apical impulse, diminished S1, physiologically split, and a soft blowing murmur in
early diastole along the right sternal border. A midsystolic click and a late systolic
murmur are noted. Abdominal examination shows a gravid uterus that is
appropriate for gestational age. Trace pedal edema and intact symmetrical pulses
are noted throughout. Fetal heart sounds are normal. An electrocardiogram shows
mild T-wave flattening. Laboratory findings include hematocrit 32% and platelet
count of 170000/?L. Fetal monitoring is instituted, and morphine is administered
for pain control.
Which of the following is the most appropriate diagnostic test?
a. Magnetic resonance angiography
b. Serum troponin measurement
c. Helical computed tomography scan
d. Transthoracic echocardiogram
e. Transesophageal echocardiography  MKSAP 13 no 7
88. A 70 year old man presents with the sudden onset of tearing chest pain. On
presentation, he has a heart rate of 130 beats/min with a systolic blood pressure
of 80 mm Hg. A bedside TEE demonstrated the presence of a proximal aortic
dissection. A pericardial effusion with partial diastolic collapse of the right
ventricle is also present. Significant respiratory variation is noted across mitral
and tricuspid Doppler inflows.
Appropriate treatment is:
a. Intra-aortic balloon pump to stabilize the hemodynamics, followed by
surgery
b. Immediate percutaneous pericardiocentesis to relieve the tamponade,
followed by surgery to replace the ascending aorta
c. Emergency angiography to define coronary anatomy, followed by surgery
d. To proceed immediately to operating room

89. A 45-years-old man presents to your office complaining of right leg pain and
swelling of 3 days duration. The patient was well until he had a wreck while riding
his dirt bike 1 week ago. The patient states that he injured his right leg in this
accident. Initially, his leg was moderately sore on weight bearing, but swelling and
persistent pain have now developed. On physical examination, you note an
extensive bruise on the patient’s right calf and 2+ edema from the foot to the mid
thigh. You suspect trauma-associated deep vein thrombosis (DVT).
Which of the following statements regarding DVT is true?
a. The post thrombotic syndrome is a rare sequel of DVT and is associated with
low morbidity
b. Thrombi confined to the calf are large and typically result in pulmonary
venous thromboembolism (VTE)
c. The most common cause of inherited thrombophilia associated with this
illness is activated protein C resistance
d. Most patients presenting with a new DVT have an underlying inherited
thrombophilia

90. A 53 year old male with a history of obesity, obstructive sleep apnea, hypertension,
and hypercholesterolemia presents to the cardiovascular clinic complaining of a
non-healing ulcer on his left ankle present for the past month. His blood pressure
is 160/80 mmHg. His physical exam is remarkable for mild bilateral lower leg
edema as well as lipodermatosclerosis and hyperpigmentation around the ankles.
A mildly tender, superficial ulceration is observed with an irregular pink base
above his medial malleolus. His feet and toes are warm, pink, and have 2-second
capillary refill and intact sensation. Laboratory test on this patient include a
random blood sugar of 160 mg/dL, creatinine of 1.1 mg/dL, phosphorus of 4.4
mg/dL.
What is the most likely the etiology of the ulceration?
a. Peripheral arterial disease
b. Critical limb ischemia
c. Diabetes mellitus
d. Chronic Venous insufficiency
e. Calciphylaxix

91. An 80 year old patient of yours is schedule to undergo total knee replacement. He
is in excellent health, and except for osteoarthritis, his medical history is not
significant. The orthopedic surgeon asks you for advice regarding VTE prophylaxis.
What would you advise for this patient?
a. Intermittent pneumatic compression devices are contraindicated because of
location of the surgery; LMWH is preferable
b. Aspirin, 325 mg q.d., should be started immediately aftersurgery
c. LMWH is contraindicated because of the riskof bleeding; intermittent
pneumatic compression devices would be preferable
d. LMWH and intermittent pneumatic compression devices are equally effective
in preventing VTE after knee surgery
e. The risk of VTE after knee replacement is so low to make prophylaxis
unnecessary

92. A 43-year-old man comes to the emergency room for evaluation of severe chest
pain hat awokehim from sleep. While the patient was seated, the pain gradually
resolved after 5 minutes, but recurred several minutes later. The patient reports
he has had nasal congestion, nausea, fatigue, and alow-grade fever for the past few
days. Physical examination shows a temperature of 38 C (100,4 F), heart rate
104/min, and recurrence of painwhile lyingin the left lateral decubitus position. A
three-component pericardial friction rub is noted. Laboratory findingsinclude a
totalleukocyte count 0f 11,000., an erythrocyte sedimentation rate of 55 mm/h,
and a slightly elevated troponin level. Electrocardiogram shows diffuse ST-T waves
changes.
Which of the following is the best course of treatment for this patient?
a. Colchicine, 0.5 mg twice a day orally 2 weeks
b. Prednisone, 25mg/dayorally for 2 weeks, followed by a taper
c. Indometacine, 25 mg four times a day orally for 2 weeks  MKSAP 13
d. Metoprolol, 25 mg twice a day orally
e. Reteplase, 10 U by intravenous bolus, repeated in 30 minutes

93. A 66-year-old Caucasianwomanis evaluated in the heart failure clinic for


consideration of advanced therapeutics, including possible heart transplantation.
She has a longstanding history of ischemic cardiomyopathy with an EFof 15%
despite optimal therapy with beta-blockers, ACE inhibitors,aldosterone
antagonists,digoxin, and diuretics. She has a 60 pack-year history of smoking,
hypertension,anddiet-controlled diabetes, and had a defibrillator/biventricular
pacemaker device placed 2 years ago. Her primary cardiologist was concerned
about her worsening functional status, as she becomes dyspneic with minimal
activity. She performed “poorly” on 6MWT, primarily limited by dyspnea. A
cardiopulmonary exercise test reveals peakVO2 of 16.1 ml/kg/min.
Which of the following statements is TRUE regarding this patient and her
functional capacity?
a. The 6MWT does not effectively discriminate the etiology of a patient’s
functional limitations, and it therefore has no prognostic value in patients
with heart failure
b. Peak VO2 is affected by age and gender, both of which should be accounted
for when assessing functional capacity
c. NYHA classification is an objective assessment of functional status with little
inter-observer variability
d. Based on her peak VO2, she should be considered for heart transplantation at
this time

94. A 45 year old woman presents to the emergency department with eight hours of
chest pain. This pain is constant, severe, and mid-sternal in location. She noted that
it is worse when she lies down. She denies exposure to sick friends or relatives.
Past history is notable for tobacco use, borderline hypertension, and elevated
cholesterol.Herfather died of myocardial infarction at age 67 years. Examination is
notable for a heart rate of 104, and a blood pressureof 125/80 in both arms. Her
lungs are clear. On cardiac examination, she has a prominent friction rub with two
out of three components present. The remainderof examination is unremarkable.
Which of the following ECG abnormalities does not suggest pericarditis in the
absence of an effusion?
a. PR segment elevation in lead aVR  MKSAP 16
b. PR segment depression
c. Concave upward ST segment elevation
d. Electrical alternans
e. Sinus tachycardia

95. A 25 year old woman who is 30 weeks pregnant is referred to you because of a
murmur that was noted during her current pregnancy as wellas intermittently in
the past. The patient is asymptomatic. Physical examination shows slight elevation
of the jugular venous pressure, with an A wave. A parasternal lift is also noted. S1
is normal, and S2 is somewhat prominent, fixed, and split. A grade 2 mid-peaking
ejection systolic murmur is noted at the left sternal border.
The statement about this patient condition include all of the following EXCEPT:
a. The sinus venosus type is almost always accompanied by anomalous
pulmonary venous connections
b. In children this condition typically experience easy fatigability and exertion
dyspnea  BRAUNWALD
c. Atrial arrhythmia are uncommon in children with this condition
d. The most common presenting symptom are exercise intoleranceand
palpitation
e. A patient foramen ovale can be found in approximately 25% of health adults

96. A 2-year-old-boy is brought into the emergency room with a complaint of fever for
6 days and the development of a limp. On examination, he is found to have an
erythematous macular exanthema over his body as shown in image A, ocular
conjunctivitis, dry andcracked lips, a red throat, and cervical lymphadenopathy.
There is a grade 2/6 laboratory systolic ejection murmur at the lower left sternal
border. A white blood cell (WBC) count and differential show predominant
neutrophils with increased platelets on smear.
Which of the following is the most likely diagnosis?
a. Kawasaki Disease
b. Scarlet Fever
c. Juvenile Rheumatoid Arthritis
d. Infectious Mononucleosus
e. Rheumatic Heart Fever

97. A 15-year-old girl is being evaluated for a heart murmur. She is asymptomatic. On
physical examination, her blood pressure is 174/104 mmHg on her right arm. Her
pulses are 2+ on her upper extremities. The femoral pulses areslightly lower in
amplitude than the radial pulses. Her cardiac examination reveals a short
midsystolic murmurin the left infrascapular area. For this patient,
Which of the following is the most likely to be found on additional studies?
a. Downward displacement of the tricuspid valve annulus toward the right
ventricle apex on echocardiogram
b. Cardiomegaly and pulmonary engorgementon chestx-ray
c. Rib notching anda dilatation of the aorta on chest x-ray
d. Stenotic pulmonic valve on echocardiogram

98. A 23-years-old woman withDown Syndrome (trisomy 21) presents to establish


care. In general, she appears comfortable. There is no evidence of cyanosis. Her
cardiac examination reveals a soft systolic murmur with amidsystolic click. The
clinical diagnosis of mitral valve prolapse is made. The patient’s mother wonders if
there are any other cardiac issues.
Which congenital heart anomaly is this patient most likely to have?
a. Transposition of great vessels
b. PDA
c. Ostiumprimum defect
d. Tetralogy of Fallot
e. Pulmonary Stenosis

99. A 45-years-old woman presents with DOE and paroxysmal AF. She has a fixed split
S2 on exam with a 2/6SEM at LSB.
Echocardiogram shows:
Dilated RV and RA, normal LC
Secundum ASD
PA systolic, 70 mmHg
The following are found at catheterization:
Saturation (%): IVC 70;SVC 67; RA 82; RV 87; PA 88; LV 98; Aorta 98; FA 98
Pressure (mmHg): PA 70/50; FA 120/70, PCWP 10
TCO 6.0 L/min
What is the Qp/Qs?
a. 3.0:1.0  MAYO
b. 2.0:1.0
c. Need more information
d. 1.5:1.0
e. 2.5:1.0

100. A 25-years-old woman who is 30 weeks pregnant is referred to you because of a


murmur that was noted during her current pregnancy as well as intermittently in
the past. The patient is asymptomatic. Physical examination shows slight elevation
of the jugular venous pressure, with an A wave. A parasternal lift is also noted. S1
is normal, and S2 is somewhat prominent, fixed, and split. A grade 2 mid-peaking
ejection systolic murmur is noted at the left sternal border.
Which of the following is the most likely diagnosis?
a. Aortic valve regurgitation
b. Mitral valve stenosis
c. Physiologic murmur related to pregnancy
d. Pulmonary valve stenosis
e. Atrial septal defect with associated volume overload  MKSAP 13

101. A 32-years-old man presents to establish care. His medical history is


unremarkable. On physical examination, the patient’s blood pressure is 132/84
mmHg, and his pulse is 88 beats/min. Cardiac examination reveals a soft systolic
murmur, heard best at the left upper sternal border. The second heart sound is
split and has no variability with respirations. There is no evidence ofcyanosis.
What is the most likely diagnosis for this patient?
a. Pulmonary insufficiency
b. Bicuspid aortic valve
c. Ostiumsecundum defect
d. Tetralogy of Fallot
e. Coronary arteriovenous fistula

102. A 20-year-old woman who has a family history of MarfanSyndrome comes to you
for pre pregnancy counseling. Echocardiogram and CT scan of the chest show 42-
mm dilation of the aortic sinuses. The patient is asymptomatic and does not have
mitral valve prolapse.
Which of the following is the most appropriate recommendation?
a. Avoid pregnancy because of the autosomal dominant nature of Marfan
Syndrome
b. Avoid pregnancy at this time because of the size of the aorta, and initiate
treatment with a ?-blocker  MKSAP 13 no 83
c. Proceed with pregnancy, preferably sooner rather than later because of the
size of the aorta
d. Recommend urgent aortic surgical intervention now
e. Initiate treatment with a ?-blocker, and proceed with pregnancy

103. You are asked to review an ECG of a baby on the intensive care unit. The baby was
well at birth, but soon became unwell and cyanosed. There was no heart murmur.
ECG findings reveal a superior axis, absent right ventricular voltages, and a large P
wave.
What is the MOST likely diagnosis?
a. Tricuspid atresia  MYUNG K PARK
b. Total anomalous pulmonary venous connection (TAPVC)
c. Transposition of the great arteries
d. Complete atrioventricularseptal defect
e. Critical pulmonary stenosis
104. Sick sinus syndrome in post op Congenital Heart Disease occurs most often in:
a. TOF repair
b. None of the above
c. Arterial switch  MYUNG K PARK
d. Fontan and Gen

105. 10 day-old baby boy is admitted to PICU with severe cyanosis. The parents and
midwives have noted the appearance of their baby since he was born. The parents
did not bring the baby the tertiary hospital because they refused to be referred. In
the last 24 hours the baby looked very sick and fatigue so that finally he was
brought to our hospital. On physical examination the baby is cyanotic with poor
periphery perfusion. He was intubated and saturating at 45%. He got a normal S1
with enhancement of S2. Continuous murmur grade 3/6 heard ICS 2 LSB. No gallop
is heard. On chest x-ray the lungs is oligemic.
Which of the following is a possible diagnosis differential:
a. Edward Syndrome
b. Ventricle Septal defect
c. AP window
d. Patent DuctusArteriosus
e. A lesion of Duct dependent pulmonary circulation

106. Based on epidemiological studies, which of the following risk factors has the
lowest relative risk range for developing lower extremity peripheral arterial
disease?
a. Diabetes
b. Hypercholesterolemia
c. Smoking
d. Hyperhomocysteinemia Postulat Heri Hernawan
e. Hypertension

107. A 52-year old woman is referred for shortness of breath. Her clinical examination
show a 2/6 diastolic murmur along the LSB and a wide pulse pressure. The patient
has no sign of heart failure but has a third heart sound and soft systolic murmur of
MR.In patients with a barely audible diastolic murmur and heart failure, what sign
is suggestive that severe AR is the cause of the heart failure?
a. A BP of 130/45 mmHg  MAYO
b. A murmur of functional MR
c. A third heart sound
d. An increased second heart sound
e. A decreased first heart sound

108. A 25 year old woman presents with exertional dyspnea and orthopnea in the 30th
weak of her first pregnancy. She has a history of rheumatic fever in childhood and
has not a recent cardiac evaluation. She is currently on no medications. Physical
examination reveals a pulse of 100 bpm with regular rhythm. The BP is 110/76
mmHg. There is mild JVD. A and V wave are visible. The lungs are clear. Cardiac
examination reveals a palpable first heart sound and a parasternal lift.
The second heart sound is somewhat increased. There is an opening snap followed
by a grade 2/6 diastolic rumble note at the apex and LSB. The ECG demonstrates
sinus rhythm with LA abnormality. A TEE is performed and this demonstrates MS.
Which of the following is most appropriate at this time?
a. Echocardiography hemodynamic study
b. MVR
c. Institution of HR control, diuresis, and warfarin  MAYO
d. Open mitral commissurotomy
e. PMBV
109. The following data were obtained from 75 year old man with a calcified aortic
valve: left ventricular outflow tract (LVOT) velocity (V1) 0.8 m/s, transaortic
velocity (V2) 4m/s, LVOT diameter 2.
True statement about this case include all the following EXCEPT:
a. Syncope may have been due to an arrhythmia in this patient
b. Orthopnea, paroxysmal nocturnal dyspnea, and pulmonary edema are late
manifestations of this disorder
c. Gastrointestinal bleeding has been associated with this disorder
d. Patient with this disorder who describe angina may not have significant
coronary arterial obstruction
e. Syncope in this disorder commonly occurs without significant change in
systemic vascular tone  BRAUNWOL

110. A 30 year-old bank executive is referred to outpatients for assessment by GP. He is


known to be hypertensive. On routine follow-up he has been found to have a
systolic and diastolic murmur. He is usually very fit and well and on the advice of
his GP has recently taken up swimming which he finds helpful for a long-standing
complaint of back pain. He is now swimming up to 30 lengths of the swimming
pool twice a week and feels well.
The GP has performed some routine blood examination including FBC,
electrolytes; these are normal. Current medication comprises amlodipine 5 mg
daily and lisinopril 20 mg daily. On examination pulse 70 bpm; blood pressure
200/100 mmHg; JVP not elevated; S2 is normal; added S4.
On auscultation in outpatients an ejection systolic murmur radiating to the carotid
is heard. There is an early diastolic murmur heard at the left sternal edge, and mid-
to-late-diastolic murmur at the apex.
All of the following are indication for surgery in pure aortic regurgitation, EXCEPT?
a. LV systolic dimension greater than 5 cm
b. Progressive LV dilation (>0.75 cm over 12 months)  UNCOVER
c. Symptomatic AR
d. Falling ejection fraction
e. LV end diastolic dimension greater than 7 cm

111. The echocardiography was performed the pressured half time of the mitral valve
was measured at 440 ms. MVA for this patient:
a. 1.1 cm2
b. 2.0 cm2
c. 0.5 cm2
d. 1.5 cm2
e. Cannot calculate with
information available

112. A 25 year old woman presents with exertional dyspnea and orthopnea in 30th
weeks of her firs pregnancy. She has a history of rheumatic fever in childhood and
has not had a recent cardiac evaluation. She is currently on no medications.
Physical examination reveals a pulse of 100 bpm with regular rhythm. The BP is
110/76 mmHg. There is mild JVD. A and V wave visible. The lungs are clear.
Cardiac examination reveals a palpable first heart sound and a parasternal lift. The
second heart sound is somewhat increased. There is an opening snap followed by a
grade 2/6 diastolic rumble note at the apex and LSB. The ECG demonstrates sinus
rhythm with LA abnormality. A TEE is performed and this demonstrates MS
The patient is started on medical therapy. She return with persistent symptoms of
dyspnea and orthopnea after 1 wk of therapy. Physical examination demonstrates
a HR at 65 bpm. The cardiac examination findings are similar to those previously
noted. A limited TTE is repeated. This demonstrates similar mitral valve
morphology. The resting mean gradient across the mitral valve is 12 mmHg. The
calculated valve area is 1.0 cm2. The calculated RVSP is 60 mmHg.
Which of the following is the most appropriate at this time?
a. Change medical therapy
b. PMBV  MAYO
c. Urgent cesarean delivery
d. Open mitral commissurotomy
e. MVR

113. A 63-year-old man is evaluated for a rash. During evaluation, you note a grade 3/6
holosystolic murmur at the apex that radiates throughout the precordium and is
loudest along the left sternal border. The left ventricular point of maximum
impulse is diffuse and displaced laterally. The heart rate is 94/min and blood
pressure is 136/80 mmHg. Jugular venous pressure is increased. The rest of the
findings on physical examination are unremarkable, except for an area of contact
dermatitis under the patient’s wristwatch band.
The patient works full-time as dairy farmer and has not seen a physician for 10
years. He denies shortness of breath, orthopnea, or paroxysmal nocturnal dyspnea.
Electrocardiogram shows sinus rhythm with left atrial enlargement.
Echocardiography shows myxomatous degeneration of the mitral valve, with
partial flail of the posterior leaflet and severe mitral regurgitation. The left atrium
is enlarged. The left ventricle is also enlarge, measuring 71 mm in diastole (normal
range, 37 to 57 mm) and 51 mm in systole (normal range for fractional shortening,
28 mm to 44 mm). The left ventricular ejection fraction is estimated at 52%
(normal > 55%).
Which of the following statements about surgical intervention for this patient is
true?
a. Surgical is indicated despite the absence of symptoms.  MKSAP 13 no 5
b. Surgery is indicated if the findings on myocardial perfusion imaging with
sestamibi are normal
c. Surgery is contraindicated based on the presence of left ventricular systolic
dysfunction
d. Surgery is indicated if the left ventricle remains dilated after 8 weeks of
afterload-reducing therapy with ACE inhibitor
e. Surgery is indicated if transesophageal echocardiographic imaging shows
that the mitral valve is amenable to repair.
114. A 21-year-old intravenous drug abuser came to the hospital with fever and malaise
of 3 weeks duration. His chest x-ray showed right-sided pneumonia and bilateral
effusions. The initial examination was remarkable for inspiratory wheezes and
dullness at the right base. There were prominent c-v waves in his jugular venous
pulse and a hyperdynamic precordium was noted. A Grade 3/6 systolic murmur
was audible most prominently along the left sternal border. Five of six blood
cultures were positive for Pseudomonas aeruginosa and he was started on
tobramycin and piperacillin. However, after 3 weeks he had persistent fever to
43oC and I+ pedal edema, and blood cultures were again positive for
Pseudomonas.
Appropriate therapy at this time would be:
a. Tricuspid valvulectomy with replacement by a Hancock porcine prosthesis
and antibiotic for 4 weeks
b. Tricuspid valvulectomy with replacement by a St. Jude mechanical prosthesis
and continued antibiotics for 4 weeks
c. An additional 4 weeks of treatment with higher doses of tobramycin and
piperacilin
d. Antibiotic changed to tobramycin and ceftazidime and given for additional 4
weeks
e. Tricuspid valvulectomy with replacement by a prosthetic valve and antibiotic
treatment for 4 weeks.

115. A 30 year-old bank executive is referred to outpatients for assessment by GP. He is


known to be hypertensive. On routine follow-up he has been found to have a
systolic and diastolic murmur. He is usually very fit and well and on the advice of
his GP has recently taken up swimming which he finds helpful for a long-standing
complaint of back pain. He is now swimming up to 30 lengths of the swimming
pool twice a week and feels well.
The GP has performed some routine blood examination including FBC,
electrolytes; these are normal. Current medication comprises amlodipine 5 mg
daily and lisinopril 20 mg daily. On examination pulse 70 bpm; blood pressure
200/100 mmHg; JVP not elevated; S2 is normal; added S4.
On auscultation in outpatients an ejection systolic murmur radiating to the carotid
is heard. There is an early diastolic murmur heard at the left sternal edge, and mid-
to-late-diastolic murmur at the apex.
On echocardiogram is performed and reported as follows: LV non-dilated; severe
LVH; normal LV systolic function; LA mildly dilated at 4.2 cm; aortic root 4.0 cm;
aortic valve thickened (bicuspid?), aortic peak gradient 20 mmHg, moderate AR;
normal mitral valve; normal right heart.
Based on this echocardiogram interpretation, what is the most likely etiology of
aortic regurgitation?
a. Coarctatio aorta  UNCOVER CASE 12
b. Endocarditis
c. Marfan’s syndrome
d. Ankylosing spondylitis
e. Aortic dissection

116. A 29-year-old Papua woman is evaluated because of severe substernal chest pain.
She has not had previous medical care, but has no known medical problems and
takes no medication. Over the past 2 months, she has noted progressive fatigue,
dyspnea, and palpitations. Two hour ago, she noted the onset of severe 9/10
substernal chest pain radiating to the jaw with moderate shortness of breath. Her
blood pressure is 110/90 mm Hg and her heart rate is 110/min and irregular.
On examination, she has jugular venous distention. She has crackles one third of
the way up both lung fields. There is a parasternal heave. She has a rapid
irregularly rhythms, accentuated P2, and II/VI early diastolic rumble at the apex.
Chest radiograph show right ventricular and left atrial enlargement with moderate
pulmonary edema.
Her electrocardiogram show atrial fibrillation with rapid ventricular response,
right bundle branch block, right axis deviation, and 3-mm ST segment elevation in
lead V2-V5.
Which of the following is the most likely cause of this patient chest pain?
a. Coronary atherosclerosis with plaque rupture
b. Coronary thromboembolism from left atrial thrombus MKSAP 13 no 80
c. Coronary thromboembolism from a left ventricular thrombus
d. Coronary vasospasm
e. Coronary arteritis

117. A 28-year-old woman has palpitation that she describes as a “heavy beat”
associated with a pause. They have occurred sporadically over the past several
years, but have become more frequent in the last few months. She has no
associated lightheadedness or syncope. Heart rate is 72/min and blood pressure is
108/68 mmHg. Physicalexamination show a mid-systolic non-ejection click. A
Surface 12-lead electrocardiogram shows sinus rhythm with a normal axis and
intervals and no evidence of ventricular pre-excitation. Transthoracic
echocardiography shows posterior leaflet mitral valve prolapse with mild late-
systolic mitral regurgitation. Left atrial size and left ventricular size and systolic
function are normal. A 24-hour Holter monitor shows a total of 728 isolated,
unifocal premature ventricular complexes without couplets or ventricular
tachycardia.
Which of the following is the most appropriate next step in the management of this
patient?
a. Initiate treatment with lisinopril 10 mg/dl
b. Initiate treatment with propafenone
c. Order transesophageal echocardiography
d. Order another echocardiogram in 6 months
e. Obtain follow-up history and physical examination in 1 to 2 years  MKSAP
13 no 75

118. The following data were obtained from a 75 year old man with a calcified aortic
valve: left ventricular outflow tract (LVOT) velocity (V1) 0.8 m/s, transaortic
velocity (V2) 4 m/s, LVOT diameter 2 cm. The calculated aortic valve area (AVA)
is:
a. 0.4 cm2
b. 0.8 cm2
c. 1.2 cm2
d. 1 cm2
e. 0.6 cm2

119. A 30 year-old bank executive is referred to outpatients for assessment by GP. He is


known to be hypertensive. On routine follow-up he has been found to have a
systolic and diastolic murmur. He is usually very fit and well and on the advice of
his GP has recently taken up swimming which he finds helpful for a long-standing
complaint of back pain. He is now swimming up to 30 lengths of the swimming
pool twice a week and feels well.
The GP has performed some routine blood examination including FBC,
electrolytes; these are normal. Current medication comprises amlodipine 5 mg
daily and lisinopril 20 mg daily. On examination pulse 70 bpm; blood pressure
200/100 mmHg; JVP not elevated; S2 is normal; added S4.
On auscultation in outpatients an ejection systolic murmur radiating to the carotid
is heard. There is an early diastolic murmur heard at the left sternal edge, and mid-
to-late-diastolic murmur at the apex.
How would you assess clinically the severity of chronic aortic regurgitation in this
patient if you establish this patient has aortic regurgitation component? Choose
the false one
a. Collapsing pulse
b. Low diastolic pressure
c. Degree of LV dilatation
d. 4th heart sound as clinical feature of heart failure  UNCOVER case 12
e. Length of the diastolic murmur

120. Which of the following disruption of normal myocyte signaling occurs in heart
failure?
a. Internalization of the ryanodine receptor, making it unavailable on the
surface membrane.
b. Increase in the speed of cross-bridge cycling
c. Use of T-type rather than L-type calcium channels to initiate contraction
d. Increase levels of SERCA2a with calcium overload of the sarcoplasmic
reticulum
e. Uncoupling of the beta-1 adrenergic receptors from G proteins  Circulation
review

121. A 30 year-old bank executive is referred to outpatients for assessment by GP. He is


known to be hypertensive. On routine follow-up he has been found to have a
systolic and diastolic murmur. He is usually very fit and well and on the advice of
his GP has recently taken up swimming which he finds helpful for a long-standing
complaint of back pain. He is now swimming up to 30 lengths of the swimming
pool twice a week and feels well. The GP has performed some routine bloods
examination including FBC, electrolytes; these are normal. Current medication
comprises amlodipine 5mg daily and lisinopril 20 mg daily. On examination pulse
70 bpm; blood pressure 200/100 mmHg; JVP not elevated; S2 is normal; added S4.
On auscultation in outpatients an ejection systolic murmur radiating to the carotid
is heard. There is an early diastolic murmur heard at theleft sternal edge, and mid-
to-late-diastolic murmur at the apex.Based on these findings there are some
differential diagnosis can be established, and the most likely diagnosis is?
a. Ruptured sinus of Valsava
b. Pure aortic regurgitation  UNCOVER Case 12
c. Mixed aortic valve disease
d. Aortic regurgitation with ruptured sinus of Valsava
e. Aortic regurgitation with mitral stenosis

122. A 25 years old pregnant woman (G1P0A0, gestational age 32 weeks) was admitted
to the hospital with main complain shortness of breath and orthopnea. She never
complained these symptoms before. Patient also complains hemoptysis. On
physical examination, blood pressure 100/60 mmHg, Heart rate was 132 bpm
irregularly irregular, Respiratory rate 28 breaths/minute. Patient has distended
jugular vein and left parasternal heave. Cardiac auscultation revealed low pitch
rumbling mid diastolic murmur. Chest X-ray revealed congested upper lobe vein
and Kerley B lines.
Below is predictive factors of successful intervention for this patient, EXCEPT:
a. Good mobility of the valve
b. Mild calcification
c. Mild mitral regurgitation
d. Minimal subvalvular disease
e. Absent of congestive heart failure
123. A 25-years-old pregnant woman (G1P0A0, gestational age 32 weeks) was
admitted to the hospital with main complain shortness of breath and orthopnea.
She never complained these symptoms before. Patient also complains hemoptysis.
On physical examination, blood pressure 100/60 mmHg, Heart rate was 132 bpm
irregularly irregular, Respiratory rate 28 breaths/minute. Patient has distended
jugular vein and left parasternal heave. Cardiac auscultation revealed low pitch
rumbling mid diastolic murmur. Chest X-ray revealed congested upper lobe vein
and Kerley B lines.
Which one of the following is the most likely anatomic diagnosis for this patient?
a. Aortic regurgitation
b. Tricuspid stenosis
c. Aortic stenosis
d. Pulmonary stenosis
e. Mitral stenosis

124. A 25-years-old pregnant woman (G1P0A0, gestational age 32 weeks) was


admitted to the hospital with main complain shortness of breath and orthopnea.
She never complained these symptoms before. Patient also complains hemoptysis.
On physical examination, blood pressure 100/60 mmHg, Heart rate was 132 bpm
irregularly irregular, Respiratory rate 28 breaths/minute. Patient has distended
jugular vein and left parasternal heave. Cardiac auscultation revealed low pitch
rumbling mid diastolic murmur. Chest X-ray revealed congested upper lobe vein
and Kerley B lines.
Which one of the following is the indication for surgery or repair in this patient?
a. Valve area >1.5 cm2
b. Pulmonary artery systolic pressure <60 mmHg during exercise
c. Thromboembolic event
d. Pulmonary artery systolic pressure <50 mmHg at rest
e. Hemoptysis

125. A 25-years-old pregnant woman (G1P0A0, gestational age 32 weeks) was


admitted to the hospital with main complain shortness of breath and orthopnea.
She never complained these symptoms before. Patient also complains hemoptysis.
On physical examination, blood pressure 100/60 mmHg, Heart rate was 132 bpm
irregularly irregular, Respiratory rate 28 breaths/minute. Patient has distended
jugular vein and left parasternal heave. Cardiac auscultation revealed low pitch
rumbling mid diastolic murmur. Chest X-ray revealed congested upper lobe vein
and Kerley B lines.
Which of the following is the most appropriate treatment to control the heart rate?
a. Digoxin
b. Methyl Dopa
c. Verapamil
d. Amiodarone
e. Flecainide

126. A 30 year-old bank executive is referred to outpatients for assessment by GP. He is


known to be hypertensive. On routine follow-up he has been found to have a
systolic and diastolic murmur. He is usually very fit and well and on the advice of
his GP has recently taken up swimming which he finds helpful for a long-standing
complaint of back pain. He is now swimming up to 30 lengths of the swimming
pool twice a week and feels well. The GP has performed some routine bloods
examination including FBC, electrolytes; these are normal.
Current medication comprises amlodipine 5mg daily and lisinopril 20 mg daily. On
examination pulse 70 bpm; blood pressure 200/100 mmHg; JVP not elevated; S2 is
normal; added S4. On auscultation in outpatients an ejection systolic murmur
radiating to the carotid is heard. There is an early diastolic murmur heard at
theleft sternal edge, and mid-to-late-diastolic murmur at the apex.
It is important to exclude endocarditis as a cause of this patient’s deterioration.
You would askabout of the following. EXECPT?
a. Recent dental abscess
b. Skin rashes
c. The history of hematuria
d. Non-hemorrhagic stroke previously  UNCOVER
e. Weight loss, malaise or anorexia

127. An 7-year old Woman is admitted via ambulance to emergency room. She is very
dyspnea and unable to give history. Medications found with her are as follows:
furosemide 80 mgpood, atenolol 50 mg po od, warfarin, digoxin 0,123 po od.
Routine observation are as follows: Temperature 37oC, pulse 130 bpm irregularly
irregular, blood pressure 190/100 mmHg, respiratory rate 40 breaths/minute, O2
saturations are 88% on 10 L/min O2 through rebreathe mask. Examination
reveals central cyanosis and cool peripheries. Auscultation of the chest reveals
widespread inspiratory crepitation; pulses are absent below the femoral arteries
in both legs. The following investigations are available: arterial blood gas, pH 7,12;
pO2 5.8 kPa; pCO2 3,2 kPa; bicarbonate 6,0 mmol/L. Routine electrolytes; sodium
130 mmol/L; potassium 5.5 mmol/L; creatinine 3.3 mg/L; glucose 6.0 mmol/L.
Her CXR shows cardiomegaly, air shadowing and Kerley B lines. EKG recording
demonstrate widespread ST depression and ST elevation in lead aVR.
If you want to send this patient surgeon to CABG, there are some factors contribute
to the preoperative risk for operation.Angiogram is performed.
After discussed with the cardiac surgeon, he reluctant to perform operation. The
reason not to do it because of:
a. Renal disease
b. Recent acute coronary syndrome
c. Female gender
d. Impaired LV function  UNCOVER, EURO score
e. Recent NSTEMI with critical myocardial ischemia
128. A 64-year old woman is evaluated for acute dyspnea 3 days after discharge
following an inferior myocardial infarction. When she was hospitalized, urgent
coronary angiography showed single-vessel coronary artery disease with
occlusion of her mid-right coronary artery. She underwent successful stenting of
her right coronary artery, and was discharged on her third hospital day. Her
ejection fraction was 50% with inferior wall hypokinesis before discharge. The
patient’s dyspnea began 30 minutes ago. On physical examination, her pulse rate is
110 bpm, respiration rate is 34.min, and blood pressure is 100/60 mmHg. Jugular
venous pressure is elevated at 10 cmH2O, crackles are heard halfway up both lung
fields, a parasternal lift is appreciated, and there is a new grade 3/6 systolic
murmur at the left sternal border with an s3 gallop. The electrocardiogram shows
sinus tachycardia with Q waves and T waves inversions in lead II, III, aVf; and is
unchanged from the discharge electrocardiogram.
A pulmonary catheter is placed, which shows the following:
Pressure (mmHg) Oxygen Saturation (%)
Right Atrium 12 (normal 2-7) 49 (normal 60-75)
Right Ventricle 60/12 (normal 20-30/2-7) 78 (normal 60-75)
Pulmonary artery 60/32 (normal 20-30/10-15) 80 (normal 60-75)
P capillary wedge 24 (normal <14) 98 (normal >93)
Which of the following is the most likely diagnosis?
a. Recurrent myocardial infarction
b. Ventricular septal defect  MKSAP 13 no 101
c. Papillary Muscle Rupture
d. Atrial Septal Defect
e. Pericardial tamponade

129. A 70 year old farmer present with 3 days of intermittent chest pressure and
dyspnea with minimal exertion. He had one episode of nocturnal dyspnea 3 days
prior. He is currently asymptomatic. His past medical history includes HTN and
hyperlipidemia. He is currently medicated with metoprolol 25 mg twice daily and
aspirin 325 mg daily.
On physical examination his BP is 140/85 mmHg and his HR is 76 bpm and
regular. His JVP is normal. His carotid upstrokes are normal and without bruits.
His lungs are clear to auscultation. His heart has a regular rate and rhythm. The
apical impulse is in thenormal location and of normal quality. The first and second
heart sound are normal. There are no murmurs or gallops appreciated. The
abdomen is soft with no masses or bruits. The extremities have no clubbing,
cyanosis, or edema, and the peripheral pulse are normal.
The ECG shows nonspecific ST-T wave changes without frank elevation or
depression. The chest X-ray is interpreted as normal. CBC, electrolytes, and cardiac
biomarkers are all negative.
The next best step is :
a. Pharmacologic stress testing
b. Start therapy with tirofiban 0,1 mcg/kg/min
c. Increase beta blockade and add nitrates, followed by noninvasive stress
testing  MAYO
d. Diagnostic coronary angiography with possible percutaneous intervention
e. Either A or B

130. A 65 years old man admitted to hospital with sudden worsening shortness of
breath preceding by sharp chestpain and not relieved by resting. Before having
chest pain, patient was walking about 400 meters. 3 days before patient having
typical angina chest pain and diaphoresis, but patient refuse to seek medical
assistant. Patient was an active smoker, had history of diabetes and uncontrolled
hypertension. On physical examination, the patient found t have a blood pressure
of 100/70 mmHg, pulse 96 timer per minutes, His JVP were raise and found S 3/6
harsh systolic murmur at the left sternal border. Blood studies showed Hb. 14
g/dL, Ht 43,7% WBC 11900/uL, platelet 213.000/uL, CK 240, ck-mb 55.
Which of the following statement is CORRECT in this case?
a. Beta blockers is contra indicated
b. IABP has no benefit
c. Primary PCIs is the best choice for this patient
d. Hypothermic cardiopulmonary bypass with optimal myocardial protection
must be done
e. Inotropic or vasopressor must given when blood pressure is low
131. According to ACCF/AHA 2009 Focused Update of the 2005 Guidelines for the
Diagnosis and Management of Heart Failure in Adults, which of the following is
class I recommendation for the prevention of chronic HF in patients at high risk of
developing HF (stage A)?
a. Angiotensin II receptor blockers can be useful to prevent HF in patients at
high risk for developing HF who have a history of atherosclerotic vascular
disease, diabetes mellitus, or hypertension with associated cardiovascular
risk factors.
b. Routine usage of nutritional supplements to prevent the development of
structural heart disease is recommended.
c. Thyroid disorders should be treated in accordance with contemporary
guidelines.--> AHA Guidelines
d. Angiotensin converting enzyme inhibitors can be useful to prevent HF in
patients at high risk for developing HF who have a history of atherosclerotic
vascular disease, diabetes mellitus, or hypertension with associated
cardiovascular risk factors.
e. Beta blockers can be useful to prevent HF in patients at high risk for
developing HF who have a history of atherosclerotic vascular disease,
diabetes mellitus, or hypertension with associated cardiovascular risk factors

132. HF prognosis correlates MOST STRONGLY with which of the following?


a. Diastolic dysfunction
b. Resting EF
c. Impaired renal function
d. NYHA class
e. Genetic predisposition

133. A 65-years-old man admitted to hospital with sudden worsening shortness of


breath preceding by sharp chestpain and not relieved by resting. Before having
chest pain, patient was walking about 400 meters. 3 days before patient having
typical angina chest pain and diaphoresis, but patient refuse to seek medical
assistant. Patient was an active smoker, had history of diabetes and uncontrolled
hypertension. On physical examination, the patient found t have a blood pressure
of 100/70 mmHg, pulse 96 timer per minutes, His JVP were raise and found S 3/6
harsh systolic murmur at the left sternal border. Blood studies showed Hb. 14
g/dL, Ht 43,7% WBC 11900/uL, platelet 213.000/uL, CK 240, ck-mb 55.
Which of the following is the most likely anatomic diagnosis for this patient?
a. Ventricular Septal Defect
b. Ventricular Septal Rupture
c. Acute Ischemic mitral regurgitation
d. Acute myocardialinfarction
e. Acute Lung Edema

134. With regard to the diagram (figure 1).


Which of the following points or curves will
move first if a patient develops an infiltrative
process such amyloid heart disease?
a. Curve A
b. Curve B
c. Curve C
d. Curve D
e. Curve E

135. Which of the following statements is true concerning the comparison of primary
PCI and Thrombolysis in the treatment of AMI?
a. Survival is higher among elderly patients if treated with thrombolytic agents
rather than primary PCI
b. Survival with PCI in an 74-year-old diabetic female who presented 2 hours
after the onset of MI with cardiogenic shock is likely to be better than with
aggressive medical treatment including thrombolysis  MAYO
c. Success rates of rescue PCI after failed thrombolysis are similar tothose of
primary PCI
d. Survival benefit with primary PCI compared to thrombolysis is confined to
anterior MI
e. Successful PCI and survival benefit with primary PCI is not associated with
operator volume
136. After appropriate diagnostic workup and medical therapy are commenced, this
patient is found to have a 95% stenosis in the middle LAD coronary artery. This
was successfully treated with an intracoronary bare metal stent.
With regard to this patient :
a. Aspirin 81 mg plus warfarin adjusted to an INR of 2.0 to 2.5 should be
commenced
b. There is a 4% to 6% risk of stent restenosis over the next 6 months
c. Aspirin 325 mg and Clopidogrel 75 mg daily should be commenced  MAYO
d. Non invasive stress testing is required at 3 to 6 months following the
percutaneous procedure regardless of the patient’s symptom status

137. A 45-years-old man is admitted to a hospital due to typical chest pain after
exercise with 4 hours onset. He reveals that this not the first time, the symptoms
were already develop over one year and he had a syncope last month. The patient
was a heavy smoker, his father already passed away at his 50’s due to heart
disease. On physical examination the blood pressure is 140/80 mmHg, normal S1
and S2 with a grade 3/6 holosystolic murmur at the apex and axilla. An
electrocardiogram shows left ventricular hypertrophy with strain, left atrial
enlargement. Cardiac enzyme were normal.
This following were the major clinical features associated with increased risk of
SCD that could possibly happen to this patient EXCEPT?
a. Gender Guideline HOCM
b. Frequent syncope episode
c. Non sustained ventricular tachycardia
d. Family history
e. Age

138. A 58 year old man presents with atypical chest pain. Coronary angiography reveals
at 60% diameter stenosis in the mid LAD coronary artery.
Which of the following intracoronary ultrasound or Doppler measurements is the
most sensitive for determining if this intermediate stenosis is
hemodynamicalysignificant ?

a. Proximal to distal velocity ration


b. Percent plaque area in the stenosis
c. Luminal are
d. Coronary flow reserve  MAYO
e. Absolute coronary flow

139. An 80 year old hypertensive male referred to cardiology clinic with stable NYHA
FC III angina for 3 months treated with aspirin, metoprolol succinate 150 mg daily,
isosorbidemononitrate 120 mg daily, and simvastatin 40 mg daily. On exam, the
heart rate is 57, the blood pressure is 98/60 mmHg and the cardiopulmonary
exam is unremarkable. Resting ECG is within normal limits. An exercise stress test
is significant for 2 mm horizontal ST depression and exercise limiting chest
discomfort at 6 METs
The most appropriate next step would be:
a. Add calcium channel blocker
b. Cardiac catheterization
c. Increase beta blocker dose
d. Echocardiography
e. Increase nitrate dose

140. Optimal time for elective non cardiac surgery after coronary stenting with bare
metal stent:
a. Between 45 days to 180 days  CIRCULATION
b. within 45 days
c. Between 180 days to 1 year
d. Beyond 1 year

141. One year later this patient is in need of cholecystectomy. He remains


asymptomatic. the surgeon arranged for a TMT before you visited with the patient.
The patient exercised to an equivalent of 9.0 METS with a normal HR and BP
response. The ECG was interpreted as non diagnostic (<1 mm of up sloping ST
depression) at peak exercise and resolved by 3 min into recovery. At this point,
which of the following is true?
a. You should recommend IV beta blockers and IV NTG with PA catheter
monitoring and to proceed the cholecystectomy
b. Repeat coronary angiography and possible coronary revascularization will
improve the patient’s operative outcome
c. The operation should be postponed until an imaging stress test can be
obtained
d. Repeat stress testing was not necessary at this point in time as the patient
was active and asymptomatic  MKSAP 13

142. A previously healthy 36 year old African American man is admitted from the
emergency departments with newly diagnosed congestive heart failure. An
echocardiogram is obtained on admission and reveals significant four chamber
dilatation , an EF of 25%and an LV end diastolic dimensions of 7.3 cm. A left heart
catheterization reveals no evidence of obstructive coronary disease. The patient
does not use alcohol, cocaine, or other illicit drugs, and he has no recent history of
viral illness. While discussing his medical history, he notes that his father died of
heart failure at age 38. You consider the possibility that the patient has some form
of familial cardiomyopathy.
Which of the following statements is TRUE concerning familial cardiomyopathies?
a. It is estimated that < 5% of idiopathic dilated cardiomyopathies are familial
in origin
b. To identify the accurate diagnosis, all family members should be tested first
for multiple mutations associated with the disease
c. Most familial cardiomyopathies exhibit autosomal recessive inheritance
patterns
d. A thorough family history should be taken, including at least three
generations
143. A 78 year old diabetic male is referred to your clinic for preoperative evaluation
prior to left knee replacement. He currently can only walk 1 to 2 blocks before
stopping, but he is limited by knee pain and denies angina or shortness of breath.
He has an adenosine sestamibi stress test that demonstrates a small area of
ischemia at the apex. He did not note any discomfort during the test. He is
currently on 81 mg aspirin daily and has adequate beta blockade with metoprolol.
His vitals are : HR :62 bpm BP : 118/70 mmHg lipids :LDL 68 mg/gL, HDL 45
mg/dL, TG 98 mg/dL. Your next step is:
a. Proceed to coronary angiogram to define the anatomy
b. Advise patient to postpone surgery for further diagnostic testing
c. Tell patient that nothing further evaluation needed at this time  MAYO
d. Proceed to coronary angiogram to perform PCI on the LAD
e. Increase aspirin to 325 mgdaiy and continue beta blocker

144. Among patients with class II angina and one or two vessel disease, PCI is indicated
for which of the following :
a. Prevention of death
b. To improve symptoms  MAYO
c. To prevent progression of CAD
d. Prevention of MI
e. To alleviate asymptomatic ischemia

145. A 60-years-old man presented to the cardiovascular clinic for consultation. He


described typical angina climbing one flight of stairs. Symptoms was improved
with ISDN 5 mg sl. He was smoker and had history of hypertension for 5 years
which was controlled with amlodipine 5 mg od. Resting ECG was normal.
Which of the following statement about the case is correct?

a. Risk stratification with noninvasive stress test has to be done before refers
the patient to cath lab for coronaryangiography
b. Coronary angiography is inappropriate because the patient is stable and well
controlled with medicamentosa
c. Coronary angiography is appropriate for the patient without prior
noninvasive stress test
d. Coronary angiography is indicated because high risk patient

146. The channelopathy underlying the clinical presentation in patient history of


cardiac arrest is:
a. Loss of function in the potassium channel
b. Gain of function in the calcium channel
c. Gain of function in the potassium channel
d. Loss of function in the sodium channel  MAYO
e. Gain of function in the sodium channel

147. Which the following is not an AV node independent tachycardia?


a. Atrial flutter
b. Atrial fibrillation
c. Atrial tachycardia
d. Atrioventricular reentrant tachycardia

148. A 45 year old woman is evaluated for palpitations that occur intermittently during
the day, vary in severity, and cause a sensation of skipped beats. She has no other
associated symptoms. She had a similar episode 2 years ago while undergoing a
stressful job relocation, but did not seek medical attention at that time. She is now
under pressure at work, and her son is leaving for collage in 1 week. On physical
examination, her blood pressure is 160/90 mm Hg and her heart rate is 80/min.
cardiac examination shows normal heart sounds and no murmurs.
Electrocardiogram shows sinus rhythm with premature atrial contraction , and a
24-hour ambulatory monitor shows 5673 premature atrial contraction , 127
premature ventricular contractions, and no runs of arrhythmias. Results of
laboratory tests, including thyroid function tests and complete blood count are
normal. The patients remains highly symptomatic , despite reassurance.
Which of the following is the most appropriate next step in the management of this
patient?
a. Perform an exercise treadmill test
b. Start disopyramide therapy
c. Perform an electrophysiologic study
d. Start beta-blocker therapy  MKSAP 13
e. Start diuretic therapy

149. A 25 year old male with no known medical history suddenly collapsed while plaing
vigorous game of ultimate Frisbee. His friends immediately started CPR and called
118. The paramedics arrived within 5 minutes and found him in VF. He was
defibrillated successfully with one shock with return of spontaneous circulation.
He was transported to the hospital for subsequent care.
The following ECG was obtained upon arrival to the hospital: what is the most
likely diagnosis?
a. Long QT syndrome
b. Brugada syndrome  MAYO
c. Cathecholaminergic polymorphic VT
d. Timothy syndrome
e. Short QT syndrome
150. A 75-years-old women is referred urgently to the cardiology clinic. She had a
myocardial infarction 4 years earlier, percutaneous coronary intervention with a
stent for angina 12 months earlier and has two blackouts in the last month, 3
weeks apart. She tell you that in one occasion she was gardening and trying to lift a
heavy plant pot. She had no warning and suddenly found herselfon the ground. She
was alert on recovery. There was no seizure like activity. She does have exertional
breathlessness although she can manage 400m on the flat and a single fight of
stairs. She has not angina since her coronary stent 12 months earlier. Occasionally
she feels light-headed if she stand up too quickly. She is currently taking aspirin, a
beta blocker, an ACE inhibitor, a loop diuretics and a statin. Her physical
examination reveals blood pressure 130/55 mmHg, resting pulse55bpm, regular,
normal volume. The JVP is raised by 2 cm, her apex beat is displaced to the lateral
clavicular line, sixth intercostal space and there is a systolic murmur heard all over
the precordium and in the carotids. The lung field are clear and there is mild
pitting edema at the level of her shins.
The patient refuses immediate hospital admission but agrees to have a 24-hour
Holter monitor attached and is scheduled for elective day-case electrophysiology
study the following week. Coronary angiography demonstrates a chronically
occluded LAD artery, and a patent in her RCA. Programmed ventricular stimulation
is then performed , including monomorphic ventricular tachycardia at 200 bpm
with loss of consciousness and is promptly cardioverted with a single external 150
J biphasic shock.
In this case what is the most appropriate treatment?
a. Send the patient to cardiac surgeon
b. Correct the electrolyte imbalance
c. ICD implantation  UNCOVER Case 16
d. Amiodarone for preventing SCD
e. Stent implantation in occluded LAD artery
SELAMAT BEKERJA
DAN SEMOGA SUKSES

Potrebbero piacerti anche